You are on page 1of 88

Chapter 1.

Simple Interest and Simple Discount


1.1
Simple Interest
It is the interest on the amount invested or borrowed at a given rate and for a given
time.
It is usually associated with loan or investments which are short term in nature.
It is computed entirely on the original principal by simply multiplying together the
principal, rate and, time.
I=Prt
Where I interest
Pprincipal (in pesos)
r interest rate per period of time, expressed as percent or a fraction
ttime (in years) between the date the loan is made and the date it
matures or becomes repayable to the lender
Maturity value (F) the total amount the borrower would need to pay back
a. F=P+I
b. F=P + Prt leading to F=P (1+rt)
Illustrative example: 1. Find the interest on a loan of 1,000 for one year if the interest
rate is 12%.
Solution:
P=1,000
r=12% = .12t=1 year
I=Prt
= (1,000) (.12) (1 year)
=120
If the term is 2 years, then the interest is:
P=1,000
r=12% = .12t=2 years
I=Prt
= (1,000) (.12) (2)
= 240
Exercises 1.1
1. A student loan 1500 at 3% was repaid at the end of 3 years. How much interest
did he pay?
Solution:
I = prt
I =1500(.03) (3)
I = 135.00
2. An interest of 720 was paid on a 3000 simple-interest loan at the end of 2
years. What was the rate of interest charged?
Solution:
I=prt
720=3000(r) (2)
R=12%
3. 7000 is deposited to an account paying 6% simple interest. How much is in the
account after 5 years?
Solution:
I=prt
I=7000(.06) (5)
I=2100
After 5 years i+p
2100+7000=9100.00
4. A company has issued a 5-year loan of 750000 to a new vice president to
finance a home improvement project. The terms of the loan are to be paid back

in full at the end of 5 years, with simple interest at 6%. Determine the interest
which must be paid.
Solution:
I=prt
I=750000(5) (.06)
I=225000.00
5. A student has received a 150000 loan from a wealthy aunt in the United States
in order to finance his 4-year college course. The term is that the student is to
repay his aunt in full at the end of 10 years with a simple interest of 2% per year.
How much should he repay his aunt after 10 years?
Solution:
I=150000(10) (.02)
I=30000
After 10 years 30000+150000=180000.00
6. What principal will yield 201.60 interest in 3 years at 16%?
Solution:
I=prt
201.60=p (.16) (3)
P=420.00
7. What principal will accumulate to 135500 in 2 years at 15% simple interest?
Solution:
I=prt
135500=p (2)(.15)
P=104230.77
8. How many years are needed for 5700 to accumulate to 6555 at 19% simple
interest?
Solution:
I=prt
5700=6555(.19) (t)
T=1 year and 6 months
9. How long will it take for the money to double itself at 10% simple interest?
=10 years
10.Mr. Santos plans to buy a TV set from Rollie Company, which is offered for 6500
cash or 6700 if purchased at the end of 3 months. If money is worth 5% simple
interest per month, how much would he save by borrowing the money and
paying cash?
Solution:
I=prt
I=6500(.05) (3/12)
=81.25- interest expert
6700-6500=200-81.25
118.75 savings
11.An interest of 516 was earned in 8 months on an investment at 12%. How much
was invested?
Solution:
I=prt
516=p (.12)(8/12)
=6450.oo
12.How many months are needed to earn 52.50 if 1500 is invested at 14%?
Solution:
I=prt
52.50=1500(.14) t

=3 months

13.Find the maturity value on a 50000 loan at 14 4 % for 7 months.


Solution:
I=prt
I=50000(.1425) (7/12)
I=4156.25
Maturity value =p + i
=50000+4156.25
=54156.25
14.Find the interest on a 6000 loan at 17% for 4 months.
Solution:
I=prt
I=6000(.17) (4/12)
I=340.00
15.Mary Santos bought a new car. To pay for the car, which was priced at 820000,
she gave 400000 as a down payment and agreed to pay the balance in 5
months. Find the amount of interest paid if interest rate is 10.25%.
Solution:
820,000-400,000=420000
I=prt
I=420,000(.1025) (5/12)
I=17937.50
1.2
Ordinary and Exact Interest
Ordinary Interest It is computed when the term of the loan is given in days.
It is 360 days in one year
Io = Pr

( 360D )

Exact Interest 365 days in one year


Ie = Pr

( 365D )

Where D is the number of days of a given term.


Illustrative Example:
1. Find the ordinary and exact interest on 15,000 if it is invested at 12% for 60 days.
a. Ordinary Interest
P= 15,000 r=12%
D=60 days
Solution:
Io = Pr

( 360D )

= (15,000) (.12)
= (15,000) (.12)
=300

60
( 360
)
( 16 )

b. Exact Interest
P=15,000 r=12% = .12D=60 days
Solution:
Ie = Pr

( 365D )

= (15,000) (.12)

60
( 365
)

=295.89
1.3 Approximate and Exact Time
Illustrative Example:
1. On May 4, 2004 Julie borrowed 22, 000 at 10 % interest. Interest and principal
were due on September 6, 2002. What was the total amount paid by her on that
date?
P=22, 000
r=10%=.10
r=may 4 to September 6, 2002
Solution: To find an interest due on September 6, 2002, we will find first the time
between May 4 and September 6, 2002.
1. Approximate time
Year
Month
Day
2002
9
6
2002
5
4
4
2 4 months and 4 days
To find the number of days: (4 30 days) + 2 days = 120 + 2 days = 122 days.
Using approximate time, the total amount paid by Julie on September 6, 2002 can be
obtained by the following solution:

[ ( )]
[ ( )]
D

F= P 1+ r 360

122

= 22, 000 1+.10 360

= 22, 000(1.033888889)
= 22,745.56
2. Actual time
Solution:
1.
May (31-4) 27
2. September 6
249
June
30
May 4
124
July
31
125 days
August
31
September
6
125 days
To find now the total amount paid by Julie on September 6, 2002, Using actual time,
we have
F=

P 1+ r

( 360D )]

[ ( )]
125

= 22, 000 1+.10 360

= 22, 000(1.034722222)
= 22,763.89
Exercise 1.2
Exercises on Approximate and actual time
Date of investment
Date of Maturity
Actual No. of Days
1. September 4, 2003
February 12, 2003
2. August 22, 2001 March 25, 2002
3. September 18, 2003
December 12, 2004
84
4. May 14, 2000
October 10, 2002
5. November 30, 2003
March 31, 2004
119
6. May 21, 2002
September 23, 2002
123
7. October 13, 2002
March 20, 2003
156
8. June 25, 2002
August 12, 2003
9. January 6, 2004 May 20, 2004
10. October 17, 2003
April 8, 2004

Approximate No. Days


158
215

161
214
85

149

147
121
125
158

48

47
134
173

133
170

Exercise 1.2.1
Application Problems on Ordinary and Exact Interest,
Approximate and actual time
1. Find the Ordinary and exact interest if 6,500 is borrowed at 24% for 120 days.
Solution:
Io= 360

120
= 6,500 (.24) ( 360

Ie = 365

120

= 6,500 (.24) ( 365

= 520
= 512.88
2. Allan Castro borrowed 3,785 from a friend and promised to repay him in 90 days plus
12% interest. How much will he pay when the loan matures using exact interest?
Solution:
I = Prt
MV = I + P

70

= 3,785 (.12) ( 365


= 111.99

= 11.99 +3785
= 3896.99

3. On what day will 8,000 earn 180 interest when invested on April 25, 2002 at 9%
simple interest? Use ordinary interest and actual time.
Solution:
t=
=

I
Pr
180
8,000 ( .09 )

= .25
4. On September San Juan went to ABC bank to borrow 230,000 at 9% interest. Jody
repaid the loan on January 27, 2003. Assuming the loan is on exact time, ordinary interest,
how much did Jody repay on the maturity date?
Solution:
I=prt

137

MV = I + P

= 230,000 (.09) ( 360


=7,877.5

= 230,000 +7,877.5
= 237,877.5

5. In problem number 4, suppose Kaye Reyes met Jody San Juan at ABC bank and
suggested she consider the loan on exact time, exact I, recalculate the loan for Jody Under
this consumption. What is your recommendation to Jody?
Solution:
I=prt

137
= 230,000 (.09) ( 365
=7,769.59

MV = I + P
= 230,000 + 7,769.5
= 237,769.59

in exact interest and exact

time

6. Borris buying a van from the interest at his time deposit in a bank. His April monthly
interest at 12% was 125,000. What was Borris principal balance at the beginning of
April? Use 365 days.
Solution:
P=

I
rt
125,000
30
12
365

( )

= 12,626,262.6

7. Using approximate time, find the exact interest on 75,000 at 8 4


2002 to May 25, 2003.
Solution:
I=prt
= 75,000 (.0825)
= 7,509.76

443
)
365

% from March 8,

8. Mark Lopez went to his bank to find out how long it will take for 10,000 to amount to
10,700 at 12% simple interest. Express answer in days.
Solution:
I=prt

t=

= 10,000 t =

I
Pr

I
Pr
=

700
10,000 ( .12 )

= 700
= 213
9. What is the exact simple interest rate using exact time if 6,000 grew to 6,168 from
February 24, 2001 to May, 2001?
Solution:

I
Pt

r=

168
83
6,000 (
)
365

= 12.31%
10. On the first day of October 2002, Albert Flores had 21,455 in an account that earns

10 2 % interest. How much interest was added to his account at the end of December of
the same year if he made no conditions or withdrawals during this time? Use the bankers
rule.
Solution:
I=prt

89
= 21, 455(.1050) ( 360

MV = I + P

=556.94

=556.94 + 21, 455


= 22,011.94

Exercise 1.3
Exact and Approximate Time between Two Dates
Date of investment
Date of Maturity
Exact No. of Days
Approximate No. Days
1. July 15, 2001
March 10, 2002
238
2. February 2, 2003
August12, 2003
191
3. April 20, 2003 November 6, 2003
200
196
4. June 5, 2003
October 12, 2003
129
5. March 28, 2003 September 25, 2003
181
6. May 8, 2003
December 10, 2004
582
7. July 6, 2003
January 4, 2004
182
8. January 17, 2003
July 8, 2003
172
9. August 24, 2003 February 28, 2004
188
184
10. Sept. 30, 2003 April 15, 2004
198
Exercise 1.3.1

235
190
127
177
572
178
171
195

Problem Solving on Ordinary and Exact Interest; Exact and Approximate


Time

1. Find the ordinary interest and the amount on 6.200 at 7 4 %for 65 days.
Solution:
I =Prt

65

= 6,200 (.0775) ( 360


= 86.76
2. What principal invested on July 6, 2003 will amount to 12,000 on December 12, 2003 if
the rate is 9.5% and exact simple interest for the exact time is used?
Solution:
P=

I
rt

12,000
= .0095 159
365

( )

= 289,970.21
3. If 15,000 is borrowed on April 20, 2003 at 6% on what day will payment of 15,600
repay the debt and interest?
Solution:
t=
=

I
Pr

600
15,000 ( .06 )

= .6667
or 240 days
4. What is the ordinary simple interest rate if 3,000 earns 90 from February 12, 2003 to
august 8, 2003?
Solution:
r=

I
Pt

90
177
3,000
360

( )

= 0.0610
= 6.10%
5. Using approximate time, find the exact interest on 5,800 at 11% from May 15, 2002 to
July 20, 2003? Solution:
I =Prt

425

= 5,800(.11)( 365
= 742.88
6. Find the ordinary simple interest on 4,200 at 8.25% using approximate time from
March 3, 2003 to June 5, 2004.

Solution:
I =Prt

452

= 4,200 (.00825)( 360


= 435.05
7. Find the exact interest and the amount on 2,500 at 9% for 40 days.
Solution:
I =Prt

452

= 4,200 (.00825) ( 360


= 435.05
8. Using the bankers rule, find the simple interest and the maturity value if 9,000 is
invested at 12% from June 12, 2002 to May 6, 2003.
Solution:
I =Prt
MV = 984 + 9,000

328

= 9,000 (.12) ( 360


= 984

=9,984

9. Using exact time, find the ordinary interest on 6,400 at 9 4


to February 20, 2004.
Solution:
Io =Prt

from January 25, 2002

391

= 6,400 (.0975) ( 360


= 677.73
10. What is the exact simple interest rate using exact time if 1,500 earns 200 from July
10, 2002 to August 20, 2003.
Solution:
r=

I
Pt

200
406
1,500
365

( )

= .1199 or 11.99%
1.4 Simple Discount
If (I) is calculated on the principal (P) at the start of the interest period, discount (D) is
calculated on the amount (F) at the end of the period.
Discount (D) is a deduction from the maturity amount (F) of an obligation allowed to
paying it currently.
Formulas:
D = Fdt, Where D= discount
To find P, use
F= amount
P= F D or P = F (1 dt)
d= discount rate
t= time or term of discount
Derived Formulas:

d=

D
Ft

t=

D
Fd

F=

and

D
dt

Example:
1. Find the present value of 2, 000 which is due at the end of 90 days at 5% simple
discount.
Given: F = 2,000 t =

1
4

Solution:
D = Fdt
= 2,000 (.05) (

d = .05
P=FD

= 2,000 25

= 25
Alternative solution:
D = F (1dt)

=2,000

= 1.975

1( .05 )

( 14 )]

= 1,975

2. Accumulate 4,200 for 2 years and 3 months at 6 2


Given: P = 4,200 t = 2
Solution:
F=

1
4

years

% simple discount.

d = .065

P
1dt
4,200

( 14 )

1( .65 ) 2

= 4, 919.47

Exercise 1.4
1. Find the present value of 5,000 due at the end of 6 months if the discount

rate is 5 4
P= FDT

%.

=5, 000( 12

)(.0575)

= 143.75
P = 5,000 143.75

2. Discount 6,500 for 120 days at 4 2 % simple discount.


P = Fdt
=5,000

P=

( 126 )( .0575)

= 143.75

3. Discount 4,250 for 1 year and 2 months at 7 2 % simple discount.

4. Accumulate 3,200 for 2 years and 6 months at 5 2 % simple discount.

5. Accumulate 3,000 for 1 year and 9 months at 6 4

simple discount.

6. If 2,500 is due on September 20, 2003, find its value on June 22, 2003 if the
discount rate is 6 %.
7. Find the discount rate if 7,000 is the present value of 7,500 which is due at the
end of 6 months.
8. If 2,421 is the present value of 2,500 which is due in 6 months, find the discount
rate.
9. Find the amount due on February 3, 2004 if the value on October 6, 2003 is 5,250

at 6 4 % simple discount.
10. How long will it take 5,850 to amount to 6,500 at simple discount rate of 5%?
11. When will 3,000 be the present value of 3,150 if the discount rate is 6%?
1.5 Promissory Notes
It is a written promise by the drawer to pay a certain amount of money to the
drawee at a specified time.
It is also a negotiable instrument which can be sold to the bank or any lending
agency at a specified discount rate.
Discounting a note cashing a note at a bank
Bank discount the bank or the lending agency collects interest in advance
Features:
1. Date of the note the date the note was written (Dn).
2. Maturity date the date the note is due (Md )
3. Date of discounting the date the note is to be discounted (Dd)
4. Face value stated amount in the promissory note or present value (P).
5. Maturity value the sum of the face value and the interest or the final amount
(F).
6. Term of the note the difference between the date of the note and its maturity
date (tn).
7. Term of discount the difference between the maturity date and the date of
discounting. If this is
given in days, the bankers rule is used (td).
8. Proceeds remaining amount after the bank discount is deducted from the
maturity value.
Formulas:

I = Pr

( days
360 )

D=Fd td
( days
360 )
days
Proceeds = F [1d ( 360 ) ]

or I = Pr tn

D = Fd

F=P+I
Md = Dn + tn
Td = Md Dd

or

Proceeds = F D or f (1d td)


Where:
r, rate of interest
d, rate of simple discount
tn, term of the note in years
td, term of discount in years
I, amount of simple interest
D, amount of simple discount
P, Principal or face value
F, final amount or maturity of the
note
Illustrative Example:
Mr. Mananquil has a five month note of 15,900

3
interest at 8 5

Required: Proceeds =?
Solution:

dated February 6, 2002 bearing

%. If she sells the note on April 10, 2002at a bank discount rate at 8%,

determine the proceeds.


Given:
FV = 15, 900
Dn = February 6, 2002
tn = 5 months

F=

Dd, date of discount


tn, term of the note
Dn, date of the note or initial date
Md, maturity date or final date
Pr, amount of proceeds, amount
payable at the discount date

P 1+ r

( 12tn )]

15,900 1+.086

( 125 )]

= 16,469.75
Md = Dn + tn
= February 6, 2002 + 5 months
= July 6, 2002
Td = Md Dd
= July 6, 2002 April 10, 2002
=187 100
= 87 days
Proceeds =

F 1d

( days
360 )]

r=8 5

% = 8.6 % = .086

Dd = April 10, 2002


d = 8% = .08

[ ( )]
87

= 16, 469.75 1.08 360


= 16, 151.33

Exercise 1.5
Problem Solving Involving Promissory Note
1. A 150day note for 12,500 dated June 3, 2002 with interest at 11% is discounted on

September 3, 2002 at 8 2 % simple discount. Find the


a. maturity date
Solution:
June 3, 2002

154

150
304 days

b. maturity value
Solution:
F=P+I
=

180/360

c. term of the discount


Solution:
I = 12,500 .11

150
360

=172.92 + 12,500
d. discount
Solution:

td

D =F d 360
=13072.92 .0850

58
360

= 179.03
e. proceeds
Solution:
P=Fd
= 13,072 - 179.03
= 12,893.89
2. A note for 9,000 dated August 12, 2002 is due on October 20, 2002 with simple
interest at 9%. If the note is discounted on August 20, 2002 at 13%, how much should the
proceeds be?

Chapter 2: Compound Interest


2.1 Finding the Compound Amount
Compound Interest is the sum by which the original principal has been increased
by the end of the contract. The original principal plus the compound interest, is called the
Compound Amount.
Annually
m=1
Semi-annually
m=2
Quarterly
m=4
Monthly
m=12

Example 1
Find the compound amount of 20,000 compounded semi-annually for 2 years at
12%.
m=2

i=

t=2 years
Solution:

12
2 =6%

n=m x t = 2 x 2 = 4

20,000.00 = original principal


+ 1,200.00 = interest for six months (20,000 x .06)
21,200.00 = new principal at the end of 6 months
+ 1,272.00 = interest for 1 year (21,200 x .06)
22,472.00 = new principal at the end of 1 year
+ 1,348.32 = interest for 1 years (22,472 x .06)
23,820.32 = new principal at the end of 1 years
+ 1,429.22 = interest at the end of 2 years (23,820.32 x .06)
25,429.54 = compound amount at the end of 2 years

F=P

(1+ mj )

mt

and

I = F P

Or
F = P (1 + i)n

Where; P=original principal


F=compound amount

I=compound interest
i=periodic rate

(i= mr )

n=number of conversion period for the whole term (n=mt)


m=number of conversion periods
t=time of term of investment which is expressed in years
j=nominal rate of interest per year.

2.2 Finding the Compound Amountwhen n is Not an Integer


Example:

Accumulate 6,500 for 4 years and 5 months at 5 2 % compounded semiannually. (using scientific calculator)

Let P = 6,500

n =mt = 2

t = 4 yrs and 5 mos I =

r
m

.055
2

(2 125 )

=2

( 5312 )

106
12

= 8.833

= .0275

m=2

r=5 2

% = 5.5% = .055

F = P (1 + i)1/n
=6,500 (1 = .0275)8.8333
=6,500 (1.0275)8.8333
=6,500 (1.27077572)
F = 8,260.04
2.3 Finding the Compound Amount at Changing Rates
During the investment term, the creditor may change the previous interest rate
which is applied to his investment especially when the debtor demands for an extension of
payments or when he can pay earlier than the specified maturity date. Hence, the
accumulated value varies with the term as the change of interest rate is applied.
The accumulated value of the principal is calculated by applying the first interest
rate to the principal for its corresponding term, then apply the next interest rate to the
amount obtained from the first interest rate and so on until the final amount reaches on
the final date.
Example :
Find the amount at the end of 13 years of 5,500 which is invested at the rate of
12% compounded quarterly in the first 4 years, 10% compounded semi-annually in the
next 7 years, and 8% effective in the remaining years.

Given:
m=4
i=.03
t=4yrs
n=16

j=12%
m=2
i=.05

j =10%
m=1
i=.08
t=7yrs
n=14
n=1

F1 = 5, 500(1.03)16
474.65(1.08)1
= 5, 500(1.604706439)
872.62
F1 = 8, 825.885

j=8%
t=1yr

F2 = 8, 825.885(1.05)14

F3 = 17,

= 8, 825.885(1.9799316)

= 18,

F2 = 17, 474.65

2.4 Finding the Present Value


The Present Value is defined as the principal which you would have invest now at a
given interest rate, so that will amount to some predetermined future sum of money. The
Compound Discount is the difference between the future value and the present value. The
formula for the present value or discounted value of an amount is given by
P = F/(1 + i)nor

P = F(1 + i)-n

D = F-P

Example:

If money can be invested at 3 2 % compounded quarterly, find the present value


of 6,500 due at the end of 2 years and 3 months.

Let F =6,500

i=

m=4

.035
4

n = mt = 4

t = 2 years and 3 mos.

r=3 2 %
so,

P = F(1 + i)-n
= 6,500 (1 + .00875)-9
= 6,500 (1.00875)-9
= 6,500 (0.92458761)

= .00875

(2 123 )

=4

( 2712 )

= 9\

P = 6,009.82
2.5 Finding the Present Value for n Not an Integer
The given problems involve an interest rate which is not found on the table and the
term which is not an integer.
Example 1
Find the present value of 7,600 due at the end of 4 years and 4 months, if money
is worth 6% compounded quarterly.
Let F = 7,600

I=

m=4

n = mt = 4

P = 7,600 (1 + .015)-

=7,600 (1.015)-

17

17

.06
4

= .015

(4 124 )

=4

( 5212 )

= 17 3

1
3

1
3

=7,600 ( 0.772580047)
P = 5,871.61
2.6 Equivalent Rates
Two interest rates are said to be equivalent if they give equal compound amounts
at the end of the same term. Generally, the nominal rate j, is equivalent to the nominal
rate j, if and only the accumulated value of any amount P at a nominal rate j, is equal to
the accumulated value of the same amount P at nominal rate j, for equal period of
investment.

Illustration:
At j1 = 8.4116% and m = 4, if P = 1,000 and t = 2 yrs, then the accumulate value is

F = 1000

(1+ 0.084116
)
4

4(2)

= 1,000 ( 1.021029)8
= 1, 181.15

Diagram:

At j2 = 8.5% and m = 2, if P =1,000 and t = 2 years then the accumulate value is

F = 1,000

(1+ 0.085
4 )

2(2)

= 1,000 (1.04205)4
= 1,181.15
Diagram:

The formula for j, is given as


J1 = m 1
The formula for w is given by
W=

(1+ mj )

2.7 Continuous Compounding


It is very important to realize that for any nominal rate j, as the conversion period
m increases, the value of its equivalent effective rate w also increases. However, as m
frequently increases, such as daily, hourly, or even infinitely, the accumulation factor
approaches the value ej where e = 2.71828. this kind of nominal rate is called an interest
rate which is compounded continuously, denoted as j c.
To find the accumulated value of any amount P at an interest rate j c compounded
F=
continuously (
also called force of interest ), use the formula
Pejc(t)
and therefore,
the formula for the present value of any amount F at a force of
P = Fe
interest jc is
jc(t)

Another equivalent relation states that the nominal rate j c, compounded


continuously is equivalent to the effective rate w, if and only if the accumulated value of
any amount P at the effective rate w is equal to the accumulated value of the same
amount P at the force of interest jc for any time t, that is shown in the working equation.
Hence:
Jc =

Or

Jc =

log(1+w)
log2.71828

log (1+w)
log e

Example:

Find the accumulated value of 50,000 if it is invested for 5 years at 10% interest
compounded continuously.
Solution:
Let P = 50,000

t = 5 years jc = 10% = 0.1

and

F=?

Then F = Pejc(t)
= 50,000e(0.1)(5)
= 50,000(2.71828)0.5
= 50,000(1.6487213)
= 82,436.06
2.8 Finding the Interest Rate
There are situation that the interest rate is unknown in the investment problem,
whereas the present value, final value, and term of investment are known. An awareness
of how fast money will bear interest will help anyone to value his own possession.
There are two ways derive a formula for the nominal rate, j, with the given
convention period, m, present value, P, final value, F, and term of investment , t.
By logarithmic Method
To find the periodic rate of interest, j, begin from the formula
F = P ( 1+ i)n
log

F = log P + n log ( 1+ 1)

n log ( 1 + i ) = log F log P


log ( 1 + i ) =

log Flog P
n

1 + i = antilog

P
( log Flog
)
n

Hence, the formula for i is given by


i = antilog

and finally, from i =

P
( log Flog
)
n

j
m , the unknown periodic rate is j = (m)(i)

by Exponential Method
Again, to find the periodic rate of interest j, begin from the formula
F = P (1 + i )n

F
P

= ( 1 + i )n

1+i=

( FP )

The formula for j is given by


J=m

( FP ) 1n

- 1]

Example:
At what rate compounded quarterly will 7,000 become 18,500 at the end of 11 years?
Solution:
Let P = 7,000

F = 18,500 n = 4 (11) = 44

Substituting the given values to the formula,


By Logarithmic Method
i = antilog

= antilog

7,000
( log 18,500log
)
44

( 4.267171733.84509804
)1
44

= antilog ( 0.00959258) 1
i = 0.02233348
Thus the unknown nominal rate is
J = 4 ( 0.02233348 )
= 0.08933392
= 8.93%

m=4

and

j=?

2.9 Finding the Time or Term of Investment


It is very important for an investor to know how long it will take his money to
accumulate into his desired amount. The term of investment or the length of time, t, for a
given amount P to accumulate into an amount, F, at a given interest rate (j, m), may be
obtained through a Logarithmic Method.
To derive the formula for time, t, begin from the formula
F = P(1 + i)n
log

( FP )=

Hence,

n log (1 + i)

n=

log

( FP )

log ( 1+i)

or

n=

log Flog P
log(1+i)

And for n = mt, the unknown time in years is therefore,


t=

n
m

Example:
On January 25,2003 Mrs Sally Lorenzo had 400,000 on a trust fund which earns
14% converted quarterly. She plans to put up a cake and pastry business as soon as the
fund contains an amount of 500,000. On what date will that amount be available?
Solution:
Let P = 400,000

F = 500,000 j = 0.14

t=?
Then from the formula
F=P

( mj )

4t

500,000 = 400,000

(1+ 0.144 )

1.25 = (1.035)4t
log 1.25 = 4t log 1.035
4t =

log 1.25
log 1.035

4t

m=4

and

0.09691001
0.01494035

t = 1.6216 years or
t = 1 year, 7 months and 14 days
2.10 Valuation of Contracts and their Comparison
Example:
15,000 is due in 7 years. If money is worth 9.5% compounded semi-annually, find:
a. Its value on the present date
b. Its value if payment will be given 3 years before its due
c. Its value if payment will be given 2 years after its due date
Solution:
a. Let F = 15,000 i =

0.095
=0.0475
2

n = 2(7 years) = 14

ILLUSTRATION

Then
x = P = 15,000(1.0475)-14
= 15,000 (0.52220804)
x = 7,813.21
b. Let F= 15,000

i = 0.0475

n = 2(3 years) = 6 and

ILLUSTRATION

Then
x = P = 15,000(1.0475)-6
= 15,000 (0.75696502)
x = 11,354.48
c. Let P = 15,000 I = 0.0475
ILLUSTRATION

Then

n = 2(2) = 4 and

x=?

x=?

and x= ?

x = F = 15,000 (1.0475)4
= 15,000 (1.20397128)
x = 18,059.57
2.11 Comparison of Values
Example:
If money is worth 8%, (m = 2), which obligation is more valuable:
a. 8,500 due at the end of 3 years without interest; or
b. 7,000 due at the end of 2 years with accumulated interest from today at
9%, (m = 4).
ILLUSTRATION

Solution:
Let P= 7,000i =

0.09
4

= 0.0225

n = 4(2) = 8

Then
F = 7,000(1.0225)8
= 7,000 (1.19483114)
= 8,363.82. . . . This is the value of obligation (b) at its maturity
date. If comparison date is at t = 2 then discount obligation (a) for n = 2(1) at

i=

0.08
2

= 0.04. Therefore,

P = 8,500 (1.04)-2
= 8,500 (0.092455621)
= 7,858.73

2.12 Equation of Values


Sum of the values of old obligations = Sum of the values of new obligations
According to the selected Focal Date, FD.
Procedure:
Step 1.

Represent the unknown with the letter x.

Step 2.

Construct the Time Diagram

a. Locate the due amounts of the set obligations on the upper part of the diagram,
b. Locate the due amounts of the set of new obligations on the lower part of the
diagram
Step 3.

Select the most convenient Comparison or Focal Date, FD. Draw arcs from

each due amount of the old and new obligations, all arcs pointing towards the Focal
Date.
Step 4.

Prepare an equation of values by adjusting the exponent of each

accumulation factor, (1 + i)n, according to the chosen Focal Date.


Example:
A debtor owes 20,000 at the end of 2 years and 50,000 at the end of 8 years. If
interest is at 7% effective rate, what single payment at the end of 6 years would
liquidate both debts?
Solution:
Let x be the single payment. Draw a time diagram and locate the two old
obligations and the new single payment.
Diagram:

The focal date may be chosen arbitrarily but for convenience, choose end of
6 years as focal date. Thus 20,000 must be accumulated for 4 years at 7%
effective rate and 50,000 must be discounted for 2 years at 7% effective rate. The
single payment x which lies at the focal date is neither accumulated nor discounted.
The equation of values may now be expressed as
New payment = Sum of Old Payments (at focal date end of 6 years)
x = 20,000 (1.07)4 + 50,000 (1.07)-2
=20,000 (1.310790601) + 50,000 (0.873438728)
=26,215.92 + 43,671.94
x = 69,887.86

Chapter 3: Simple Annuities and their Applications

3.1 Basic Concepts and Terminologies


Annuity a sequence of payments, usually, equal, made at regular intervals.
Annuity certain it is certain when the term of annuity is fixed
Contingent annuity or annuity uncertain is one for which the first and last payments or
both cannot be foretold accurately
Payment The time between successive payments whereas interval or conversion period
Periodic payment the size of each payment
Term of annuity the time from the start of the first payment to the last.
Simple annuity when the payments interval coincides with the interest conversion
period.
Ordinary annuity is one for which the periodic payment are made at the end of each
period
Annuity due when the payments are made at the beginning of each period
Deferred annuity one whose first payment is due at some later time
The following symbols will be used in dealing with ordinary annuity formulas.
R = periodic payment of the annuity
n = total number of payments
i = interest per conversion period
S = amount of annuity
A = present value of an annuity
3.2 Amount and Present Value of an Ordinary Annuity
The amount or final value, denoted by S, of an ordinary annuity is the sum of all
accumulated value of the sets of payments due at the end of the term, while the present
value of an annuity, denoted by A, is the sum of all discounted value of several payments
due at the beginning of the term.
A and S are related by the equations
A = S (1+i)-n
and S = A (1+i) n
Where,

A is the present value of S due in the periods


S is the amount of A for n periods
Illustrative example
Consider an ordinary annuity of 10,000 per year payable for 3 years with money
worth 10%. To find the amount S of the annuity, add the accumulated payments of each
period at the end of 3 years.

3.1 Basic concepts and Terminologies


An annuity is a sequence of payments, usually equal, made at regular
intervals. Some examples of annuities are quarterly payments of a lots
amortizations, premiums on insurance, periodic pensions, monthly payments on
installment purchases, among others.
Annuities are divided into: annuity certain and contingent annuity or annuity
uncertain. Annuity certain; when the term of the annuity is fixed. A contingent
annuity is one for which the first and last payments or both cannot be foretold
accurately.
The following symbols will be used in dealing with ordinary annuity formulas:
R - periodic payment of the annuity
n - total number of payments
i - interest per conversion period
S - amount of an annuity
A - present value of an annuity
3.2 Amount and Present Value of an Ordinary Annuity
The amount or final value, denoted by S, of an ordinary annuity is the sum of
all the accumulated value of the set of payments due at the end of the term,
while the present value of an annuity, denoted by A, is the sum of all the
discounted value of several payments due at the beginning of the term.
A and S are related by the equations
and
A = S(1
+ i)-n

where S =n A(1
A is the present value of S due in n periods and
+ i)
S is the amount of A for n periods
Illustrative Example
Consider an ordinary annuity of 10,000 per year payable for 3 years with
money worth 10%.
To find the amount S of the annuity, add the accumulated payments of
each period to the end of 3 years.
Term

10,000 =P10,000
10,000(1 +.10)1 = 11,000
10,000(1 +.10)2
= 12,100
S
= P33,000
To find the present value of an annuity, add theTerm
discounted payments.

10,000(1 + .10)-1 = 9,090.91


10,000(1 +.10)-2 = 8,264.46
10,000(1 +.10)-3
=
7,513.15
A
= 24,868.52

Hence,
A=
A=
A=

from the relation


S(1 + i)-n
and S = A(1 + i)n
33,100(1 + .10)-3
and S = 24,868.52(1 + .10)3
24,868.52
S = 33,100

3.3 Ordinary Annuity Formulas


The periodic of R for n periods.
Term
R
0

1
2

R
n-1 n (periods)
R
R(1 + i)1
.
.
.
R(1 + I)n-3
I)n-2
RR(1
at +
the
end of the
R(1
n-2+ I)n-1
n-1

S = sum of the accumulated values of


term
S = R + R(1 + i) + . . . R(1 + i) n-3 + R(1 + i) + R(1 + i)
multiplying (1) by (1+i)
(1 + i)S = R(1 + i) + R(1 + i)2 . . . R(1 + i)n-3 + R(1 + i)n-1 + R(1 + i)n
subtracting (1) from (2) we get
(1 + i)S S = R(1 + i)n R
iS = R[(1 + i)n 1]

R (1+ i)n1
S
S=
=R
1
n
(1+i) 1
or
i

if we letSn i =

(1+i) 1
the formula for S can be written as
i

where

Sn i =
Rsn i

S id the amount of an ordinary annuity


R is the periodic payment
j
I = m = interest rate per period
N = mt = number of payments

The symbol, Sn I, is read as S angle n at I


Sn i =
n
(1+i)
1
To derive
the
present value of an ordinary annuity we use the relation
i + i)-n
A = S(1

A = RSn i(1 + i)-n


A= R

( 1+i ) 1
i

(1 +

A= R

1(1+i)n
if we let n i =
i
n
1(1+i )
then the formula
i of A can be written as
A=
Ran i

where A is the present value of the annuity an I, is read as a angle n at i


an i =
n

1(1+i )
i
Illustrative Example
1
Find the amount and present value of an annuity of 1,500 payable for
2 years if money is worth 105 compounded semi-annually.
j
.10
Given
R = P1,500
i
=
=
j = 10% - .10
m
2 =.05
i = 2 years
n = mt = 2(2) = 4
m=2
periods

To find S
S
S
S
S

or

=
=
=
=

Rsn i
1,500svn .05
1,500(4.310125)
P6,465.19

S=R

( 1+.05) 1
i

s = 1,500

]
4

( 1+.05) 1
.05

s = 1,500(4.310125)
s = 6,465.45

to find A
A
A
A
A

=
=
=
=

Ran i
1,5004 .05
1,500(3.54595)
P5,318.93

or by using the relation


A = S(1 + i)-n
A = 6,465.19(1 + .05)-4
A = P5,318.93

Illustrative Example 2
Mrs. Cruz purchased a house and lot. If she paid 400,000 as down
payment and promised to pay P5,000 every 3 months for the next 10 years
at 15% compounded quarterly, find the cash value of the house.
Given:

DP = 400,000

j = 15%

t = 10 yrs.

R = 5,000

m=4

Solution:
The down payment is not part of the annuity. The cash value of the
house is 400,000 plus the present value of the ordinary annuity of 40
monthly payments of P5,000 each. The cash value is
Cash Value = down payment + Ra n I
= 400,000 + 5,000 40 .0375
= 400,000 + 5,000(20.5509999)
= 400,000 + 102,754.95
Cash Value = 502,754.94
3.4 Periodic Payment of an Ordinary Annuity
a.) Periodic payments of S
S
R=
s n i or R =
b.) Periodic payment
of A
A
R=
an i or R =
Illustrative Example 1
In order to have 300,000 at the end of 15 years, how much must be
deposited in a fund every 3 months if money Is worth 8% compounded
quarterly?
Solution:
Given:
S = 300,000
t = 15 years
n = 60 periods
j
8
i = m = 4 = .02

Find R:

R=

s n i

R=

300,000
s 50 .02

R=

300,000
114,05154

R = 2,630.38

Illustrative Example 2

Mr. Tuy bought a refrigerator that cost 19,500. He paid 6,000 as down
payment and the balance will ne paid in 36 equal monthly payments. Find the
monthly payment if money is 15% compounded monthly.
Solution:
A = Cash Value Down
A = 19,500 6,000 = 13,500
n = 36
i=

.15
12

Find R:

= .0125

R=

A
an i

R=

13,500
1( 1+.0125 )36
.0125

R=

13,500
28.84726737

13,500
a.36.0125

Exercise 3.1
1 Analyn bought a dining set. She paid 2,500 as down payment and promised
to pay 800 at the end of each month for one year. What is the cash
equivalent of the set if the interest rate is 10% compounded monthly?
2 Find the amount present value of an annuity of 5,240 payable at the end of
1
every 6 months for 5 years and m6 months at 5 2 converted semiannually.
3 What is the present value of 1,230 due at the end of every three months for
3
3 years and 6 months, if money is worth 9 4 % compounded quarterly?
4 An item can be bought for 25,300 as down payment and 12 semi-annual
payments of 480 each. What is the cash equivalent of the item if money is
1
worth 5 2 compounded semi-annually.
5 If Cherry will deposit P1,500 every end of each quarterly in 9 years at the
1
rate of 6 2 compounded quarterly, how much will be the amount of her
savings at the end of 5 years?

6 What is the cash value of a set of Science Encyclopedia that can be


purchased for 12,500 as down payment and 1,450 at the end of each
month for 18 months at 15% compounded monthly?
7 In a series of monthly payments of 1,580 for 10 years, what is the cash
value of an annuity if money is worth 12% compounded monthly?
8 A man deposits 12,500 at the end of every six months in a bank which
credits interest at 9% compounded semi-annually. How much will he have
after 19 years?
9 Mr. Chua purchased a car. If he paid 400,000 as down payment and
1
promised to pay 10,500 every month for 5 years at 7 2 compounded
monthly, what is the cash value of the car?
10 Find the present value if an annuity whose semi-annual payment is 2,500
3
payable for 5 years at 8 4 compounded semi-annually.
11 A man purchased a DVD player worth 8,500. He paid P2,000 cash and
agreed to make 15 monthly payments. If interest is 18% (m=12), find the
monthly payment.
12 Marga borrowed P250,00 and agreed to repay her obligation by making equal
quarterly at the end of every months for 4 years. What is the periodic
payment, if money is worth 12.5% compounded quarterly?
1
13 How much should be invested each year in a fund paying 10 2

effective to

accumulate to P250,000 in 5 years?


14 Find the monthly payments for 30 periods to discharge an obligation of
3
300,000, if money is worth 9 4 compounded monthly.
1
15 How much should one deposit monthly in a fund that earns 12 4
compounded monthly in order to have 350,000 in 3 years and 6 months?

3.5 Finding the Interest Rate of an Ordinary Annuity


These formulas are:

a
b

(1 nRS ) = 0
nR
and (n - 1)i + 6(n + 1)i + 12 ( 1 S )
(n2 - 1)i2 6(n - 1)i + 12
2

=0

Example 1
A refrigerator can be purchased for 9,000 cash down and P630 a month for
18 months. Find the interest rate charged if it is compounded monthly.
Solution:

A = 9,00 900 8,100


R = 630
n = 18
m = 12

Formula:
Since A is known

(1 nRA ) = 0
18(630)
(18 - 1)i + 6(n + 1)i + 12 ( 1 8,100 )
(n2 - 1)i2 + 6(n + 1)i + 12
2

=0

323 i2 + 114i 4.8 = 0

To find i, we use the quadratic formula for ai2 + bi + c = 0, then


b b24 ac
i=
2a
i = -114
i=

(114 )24 (323)(4.8)


2( 323)

114 138.555404
646

i = -.3909526 or .03801146

Here we get two values of i, we disregard the negative value of i. Therefore,


the periodic interest rate is
i
i
i
i

=
=
=
=

3.80% and the nominal interest rate charged is


ixm
(.03801146)12%
45.61%

Example 2:
Payments of 750 each are made every 6 months in 3 years. At what rate
compounded semi-annually will these payments amount to 5,100?
Solution:

S = 5,100
R = 750
n = mt = 2(3) = 6

Formula: Since S is known


(n2 - 1)i2 6(n - 1)i + 12
(62 - 1)i2 6(6 - 1)i + 12

(1 nRS ) = 0
(1 6(750)
5,100 ) = 0

35i2 30i + 1.4117647 = 0

To find i, we use the quadratic formula. We have


30 3024 (35)(1.4117647)
i=
2(35)
here we set two values of i
i = .887170603 or.0494722647

Since i = .049972253 (2)


j = im = .049972253

Therefore, the interest rate charged is 9.99944 compounded semiannually.


Exercise 3.2
1 in purchasing a washing machine worth P21,000, a buyer pays 3,500 cash
and agrees to make 12 monthly payments. If the monthly payment is 1,580,
find the interest rate m charge.
2 Deposit of 8,380 are made every end of six months. At what rate
compounded semi-annually will these deposits amount to P112,400 in 14
years?
3 A television set costs P10,440. It is purchased by a down payment of 4,440
and monthly payments of P 382.84 for one year and 6 months. Find the rate
of interest converted monthly. Use either interpolation or the given
derivation of i.
4 A man invests 9,375 every six months. If he has 200,000 in five years, at
what nominal rate compounded semi-annually did hi investment earn?
3.4 The Term of an Ordinary Annuity
The number of payments can be determined when the amount or present
value, the periodic payment and interest rate are given
Formula in finding the term, n is
n=

Example 1log 1 Ac
R
log (1+1)

Victoria borrows 15,000 with interest at 15% compounded quarterly. She


will discharge the debt by paying P950 quarterly.
a How many payments of 950 are required?
b How much would the final payment be if it is made the day after the last
P950 payment?
c

How much would the final payment be if it is made 3 months after the last P950
payment?

Solution:
Given: A = 15,000
R = 950
i = .0375
a Substitute the given values into the formula
Ac
log 1
R
n=
log ( 1+ i)

n=

n=
n=

15,000(.0375)
950
log ( 1+.0375)

log 1

log.407894737
log ( 1.0375)
.389451898
.015988105

n = 24.35885291

This means that there will be 25 quarterly payments of 950 and a


final irregular payment.
b To find the final payment, the 24th payment in this case.
Let x be the irregular payment to be paid on the 24th payment. Make a
time diagram using the present as the focal date.
Ordinary annuity of 24 months
0950950950 950950

. . .

1 2 3 23 24

Focal date

x(1.0375)-24 + 950 a24 .0375

= 15,000

x(.413319095) + 14,862.58 = 15,000


x = P332.47

Therefore the final 24th payment is

= 24th payment + final irregular payment


= 950 + 332.47
= P1,282.47

c To find the concluding payment


(irregular
payment
on the 25th payment) x
Ordinary
annuity
of 24 months
th
Let y be the final payment on the 25 period
0950950950 950950
1 2 3 23 24

. . .

950 950 950


0

.3 . .

950 950 950 y


22

23 24 25

ordinary annuity of 24 payments

Using the present as the focal date


y(1.0375)-25 + 950 an .0375 = 15,000
y(.39837985) + 14,862.58 = 15,000
x = 344.85

Exercise 3.3
1 Lulu borrows 240.000 with interest at 12% compounded monthly. She will
pay the debt by paying 5,300 monthly
a Find the number of regular payments to be made.
b How much will be the final payment if it is made one month after the last
5,300 payment?
2 The present value of an annuity is 270,000 whose semi-annual periodic
payment of 15,600 will be needed?
3 Margarita borrowed 500,000 and agreed to repay it in equal quarterly
installment of 18,500 with interest at 10.5% compounded quarterly. If the
first payment is due in three months after borrowing the money.
a How many full payments must the debtor make
b How much would the final payment he if it is made on the day of the last
18,500 payment?
4 A car is worth 730,000. The buyer paid 350,000 down payment and
3
promised to pay 7,660 every month, if he is charged 7 4 compounded
monthly,
a How many full payments must be made?
b How much will be the final payment?
5 A fund of 400,000 is to be created by making a quarterly deposit of
15,000. If the fund earns 12% compounded quarterly,
a How much deposits of 15,000 are required?
b How much would the final deposit be if it is made three months after the
last 15,ooo deposit?
3.4 Annuity Due
An annuity due is one for which the first payment occurs immediately. It is
also defined as an annuity in which payments are made at the beginning of
each period.
The following symbols re used in dealing with annuity due formulas:

R = periodic payment of the annuity due


n = total number of payments
i = interest per conversion period
S = amount of an annuity due
A = present value of an annuity due
3.7.1 Present Value of an Annuity Due
The present value of an annuity due, which is denoted by A, is the sum of the
present values of the payments.
Term
0
1
R
R

3
R

...

n-1 n
R

R ...

The formula in finding the present value of an annuity due is:


A = (1st payment) + (present value of remaining payments)
A =A +
Ran1 i
Example 1
Marlon purchased a car. He paid 150,000 down payment and 10,000
payable at the beginning of each month for 5 years. If money is worth 12%
compounded monthly, what is the equivalent cash price of the car?
Given:
R = 10,000

j = 12%

12
12

Down payment = 150,000 i =


m = 12
t = 5 years
n = 60

1%

A = ? Cash Equivalent - ?

Ran1 i

A =R+

A = 10,500 +

10,500601 1

A = 10,500 + 10,500 a59 1


A = 10,500 + 10,500

59

1(1.01)
.01

A = 10,500 + 466,248.18
A = 476,748,18 (Total Installment Payment)
Cash Equivalent = Down Payment + Total Installment Payments
Cash Equivalent = 150,000 + 476,748.18
Cash Equivalent = 626,748.18

3.7.2 Amount of an Annuity Due


The amount of an annuity due, which is denoted by S, is the sum of the
accumulated values of the payments at the end of the term.
0
1
R
R

3
R

...
R ...

n-1 n
R

The formula in finding the amount of an annuity due is:


S = {Value of an annuity of (n +1) payments on the last payment date} R
S = Rsn+1 iR
Example 1
If 500 is deposited in a bank at the beginning of each 3 months for 10
years and money is worth 8% compounded, how much is in the fund at the end
of 10 years?
Given:

R = P500

j = 8%

M=4

i=

8
4

t = 10 years
n = 40

S=?

S=

Rsn+1 iR

S=

500 s40+ 1 2 500

S = 500

= 2%

(1.02)411
.02

-500

S = 31,305.01 500
S = 30,805.01

Example 2
Emil invests 5,000 at the beginning of each six months. He makes his
first deposit on January 19, 2003. How much will be in his account on January
19, 2015, if money is worth 9% compounded semi-annually?
R = 5,000
m=2
t = January 19, 2003 January 19, 2015 = 12 yrs.
n = 24
j = 9%
i=

9
2

=4 2 %

S=?
S=
S=
S=

Rsn+1 iR
5,000 s

5,000 s

24+1 4

25 4

1
2

1
2

5,000

5,000

S=

5,000

13

(1.045) 1
.045

-5,000

S = 22,826.05 5,000
S = 217,826.05

3.7.3 Periodic Payment R of an Annuity Due

The following formulas are sued in finding the periodic payment R of


an annuity due: A
S
R=

1+an 1 i

R=

Sn +1 i1

Where:
A = present value of an annuity due
S = amount of an annuity due
R = periodic payment of the annuity due
i = interest per conversion period
n = total number of payments
Example 1
What equal deposits should be placed in a fund at the beginning of each
year for 15 years in order to have 1,500,000 in the fund at the end of 15
years, if money accumulates 12%?
Given:
S = 1,500,000
m=1
t = 15
n = 15

j = 12%
i = 12%
R=?

R=

S n+1 i1

R=

P 1,500,000
S 15+1 12 1

R=

P 1,500,000
S16 12 1

R=

P 1,500,000
41.75328042

R = 35,925.32

Exercise 3.4
For each problem draw a diagram illustrating the data and the solution
1 If money is worth 16% compounded quarterly, find the present value and the
amount of annuity due of 1,500 payable quarterly for 10 years.

2 An investment of 5,000 is made at the beginning of each month for 8 years


and 7 months. If interest is 12% compounded monthly, how much will the
investment be worth at the end of the term?
3 At retirement, Mr. Nolram Ocampo finds his share f a pension fund is
2,500,000. What payment will this provide at the beginning of each month
for 25 years, for him or his estate, if the fund invested at 18% compounded
quarterly?
4 Mr. Jason Raymundo agrees to pay 2,000 at the beginning of each 6 months
1
for 5 years. If money is worth 12 2 % compounded semi-annually, find a)
the present value of his debt, before he makes a payment; b) Mr.
Raymundos outstanding liability just before his 7th payment; and c) his
outstanding liability just after the 7th payment.
5 A house and lot is bought for 1,000,000 down payment and 5,000 payable
at the beginning of each month for 5 years. What is the equivalent cash price
of the house and lot, if interest rate is 14% compounded monthly?
6 On January 19, 2003, Mr. Berlindo Santos opened a saving account for his
wife with an initial deposit of 10,000 in a bank paying 6% compounded
quarterly. If Mr. Santos continues to make quarterly deposits of the same
amount until July 19, 2008, how much will be in the account at the end of the
term? (Hint: Count the time by months.)
7 A machine will be replaced 10 years from now at a cost of 750,000. How
much should the owner save at the beginning of each quarter in order to
3
replace the old machine if his savings earn interest at 8 4 % compounded
quarterly?
8 What equal deposits should be placed in a fund at the beginning of each year
for 20 years in order to have 2,500,000 in the fund at the end of 20 years, if
1
the money accumulates 10 2 %?
9 Mayette agrees to make equal payments at the beginning of each 3 months
for 12 years, to pay all interest and principal in purchasing a lot worth
1
1,200,000 cash. If money is worth 12 2 % compounded quarterly, find the
quarterly payment.
10 A loan of 200,000 will be settled by 25 equal payments, the first of which is
to be made immediately. Find the quarterly payment, if money is worth 10%
compounded quarterly.

3.8 Deferred Annuity

Is an annuity whose term does not begin until the expiration of a


specified time
Ad

...

...

Sd

2
3
d d1 d2
deferment period n periods
(d)
No payments

d+n-2

Payments are made

The formula in finding the present value


Ad

d+n-1 d+n

Ad

of the deferred annuity is

= {present value of annuity with term (d + n) periods} {present


value of annuity with term d periods}
A d =Ran+d i Rad i

Where:
A d = present value of deferred annuity
d=

value of deferment

n=

total number of payments

R=

periodic payment of the deferred annuity

i=

interest per conversion period

Example 1
A sequence of quarterly payments of 3,500 each, with the first one due
at the end of 3 years and the last of the end of 10 years. Find the present
value of the deferred annuity, if money is worth 16% compounded quarterly.
Given:
R= 3,500

Solution:
16
i=
4

j = 16%

m=4

= 4%

d = 3 x 4 = 12; 12 1 = 11
n = 7 x 4 = 28; 26 + 1 = 29
40
11

29
RRRRRRRRRRRR
Ad

2 yrs
0

...
...

RRRRR

...
...

Sd

8
9
10
Ad
1
2
3 7
d d1 d2
d+n-2 Sdd+n-1 d+n
deferment
period
n
periods
10
1
2
3(d)
d Payments
d1 d2are made d+n-2 d+n-1 d+n
No payments
deferment
period n periods
(d)
No payments

Payments are made

Ad

Ran+d iRad i

Ad

3,500 a29 +114 3,500 a11 4

Ad

3,500 a 40 4 3,500 a11 4

1(1.04)40
.04

Ad

= 3,500

Ad

= 69,274.71-30,661.67

Ad

= 38,613.04

- 3,500

1(1.04)11
.04

Example 2
Find the cash equivalent of an item that sells for 20,000 down payment
and 20 semi-annual payments of 5,500 each, the first is due at the end of 3
years, if money is worth 10% compounded semi-annually.
Given:
R = 5,500 Down payment = 20,000

Solution:
10
i=
2

j = 10%

m=2

= 5%

d = (3 x 2) 1 = 6 1 = 5
n = 20
Ad
25
5

20
R R R R R R R R R R R R R R R R R R R R R R

10

11

12

13

2 yrs

Ad

Ran+d iRad i

Ad

5,500 a20 +5 5 5,500 a5 5

Ad

5,500 a25 5 5,500 a 5 5

25

1(1.05)
.05

Ad

= 5,500

Ad

= 77,516.70 23,812.12

Ad

= 73,704.58

- 5,500

1(1.05)
.05

3.8.1 Periodic Payment R of Deferred Annuity


The formula in finding the periodic payment R of a deferred annuity is:
R=

Ad
a d+ n ia d i

Example 1
Alex borrows 100,000 with interest at the rate 12% compounded semiannually. he agrees to discharge his obligation by paying a sequence of 10
1
equal semi-annual payments, the first being due at the end of 5 2 years. Find
the semi-annual payment.
Given:
Ad

= 100,00

J
m
n
d

= 12%
=2
= 10
= 10

12
2

= 6%

Solution:
R=

Ad
ad +n i ad i

R=

P 100,000
a20+6 ia 10 6
P100,00

R=

20

1(1.06)
.06

][ ]

110
.06

R=

P 100,000
11,469921227.360087051

R=

P 100,000
4.109834169

R = 24,331.88

Exercise 3.5
For each problem, draw a line diagram illustrating the data and the solution
Find the present values of the deferred annuity which is described, at the
specific interest rate.
1 A farm costs 750,000 cash. Paolo will pay 150,000 cash and a sequence of
15 annual payments, the first due at the end of 5 years. If money is worth
12%, find the annual payment.
2 A house and lot is offered for sale 1,500,000 cash, or 500,000 cash and a
sequence of 20 semi-annual payments of 120,000 each, the first is due at

the end of 3 years, if money is worth 14% compounded semi-annually, find


the cash value of the house and lot.
3 Find the present value of a series of quantity payments of 2,500 each, the
first payment is due at the end of 4 years and 6 months, and the last at the
end of 10 years and 3 months, if money is worth 16% compounded quarterly.
4 Find the cash equivalent of a computer set that sells for 10,000 cash and a
sequence of 10 semi-annual payments of 3,500 each, the first is due at the
1
end of 3 years and 6 months, if money is worth 10 2 % compounded semiannually.
5 Jason borrows 50,000 and agrees to pay his obligation by making 15 equal
annual payments, the first is due at the end of 3 years . Find the annual
payment, if money is worth 12%.
6 Ryan obtains a loan of 250,000 with interest t 14% compounded quarterly.
He will discharge his debt by a sequence of equal quarterly payments, the
first is due at the end of 5 years and the last at the end of 10 years find the
periodic payment.
7 Find the present value of the pension of a man, now 60 years old, and who
will receive a pension of 10,000 per month for 15 years, with the first
payment to occur one month after his 65th birthday, if money is worth 18%
compounded monthly.
8 An investment in a stock market will yield no operating profit until the end of
5 years, when investor will receive P100,000 after that, he will receive
100,000 at the end of each quarter for 10 more years. Find the present
1
value of this income if money is worth12 2 % compounded quarterly,
9 On November 20, 2002 a house and lot was bought for 500,000 down and
12 quarterly payments of 50,000 each, the first is due on November 20,
2003. Find the cash value of the house and lot, if money is worth 15%
compounded quarterly.
10 Aling Nelie won 25,000,000 in Mega Lotto. She invest winnings at 16%
compounded semi-annually with the conditions that she receive 25 semiannual withdrawals starting at the end of 2 years. Find the size of the
withdrawals.
3.8.2 Value of an Annuity in an Arbitrary Date
Example 1

A sequence of 5 semi-annual payments of 2,500 each will start with a


1
payment at the end of 2 2 years. If money is worth 12% compounded semiannually, find the sum of the values of these payment at the end of a)2 years,
1
b)4 2 years, c) 6 years, d)the actual present value of the payments.
Given;

R = 2, 500
j = 12%
m=2
i=

12
2

= 6%

n=5

Time Diagram:
R
0

2
3
4
1 yr.
2 yrs.

6
3 yrs.

8
4 yrs.

10
5 yrs.

11

12 13 14
6 yrs.
7 yrs.

Solution;
a) At the end of 2 years

b) At the end of

A = 2, 500 a5 6
A2 = 2, 500

1
2

years

S = 2,500 a5 6

1( 1.06 )
.06

S = 2,500

A = 10, 530.91

( 1.06 )51
.06

S = 14, 092.73

c) To find the value of the annuity at the end of 6 yrs, accumulate a) for 4 years, or

the result of b) for 1 2

years.

Value at the end of 6 years is = 2,500( a5 6 )(1.06)8


= 16, 784.67

Value at the end of 6 years is = 2,500( a5 6 )(1.06)3


= 16,794.67
d) To find the actual present value, discount the result of a) for 2 years or the

result of b) for 4 2

years.

Actual Present Value = 2,500 )(1.06)-4

or

=8,341.47

Actual Present Value = 2,500( a5 6 )(1.06)3


=8,341.47
Exercise
For each problem in the exercise, draw a time diagram showing the payment dates,
and the beginning and the end of the term of any ordinary annuity involved.
1. A sequence of 16 quarterly payments of 5,000 each will start with a payment at the

end of 5 2

years. If money is worth 18% compounded quarterly, find the sum of

the values of these payments at the end of a) 4 years b) 8 2

years c) 10 years d)

find the actual present value of the payments.


2. In starting a business venture, the owner is granted a loan of 750,000 at the

beginning of ach 6 months for 10 years. He agrees that money is worth 12 2 %


compounded semi-annually, and that he will discharge all accumulated liability by a
single payment at the end of 15 years. What amount must he pay to settle his
obligation?
3. If the fund accumulation at 15% compounded semi-annually, how much will be in the
fund at the end of 30 years if 5,000 is deposited in it at the end of each 6 months
for the first 25 years?
4. A fund of replacement of machinery in a plant must contain 300,000 at the end of
12 years. If the fund is invested at 10.75% compounded quarterly, what equal
deposits should be placed in the fund at the end of each 3 months just for the first 7
years?
5. The first payment of 10,000 by Paul is due in 2 years. Payment continues every 3
months until the 15th year. If money is worth 20% compounded quarterly, find the
annuity value at the beginning of the first year.

CHAPTER 4
Amortization and Sinking Fund
Amortization
Amortization is a means of repaying a debt by a series of equal time interval.
the periodic payments form an annuity in which the present value is the principal
of an interest-bearing debt. Hence we use the following annuity formulas;
A=R

1(1+i)
i

R=

Where:

Ai
1(1+i)

A = principal
R = periodic payment
i = interest per period
n = total number of payment periods

Examples:
1. An obligation of 21,000 with interest of 8% compounded semi-annually
must be paid at end of every 6 months for 4 years. a) Find the size of periodic
payment. b) Find the remaining liability just after making the 5th payment. c)
Prepare the amortization table.
Solution:

A = 21, 000
j = 8%

a) R =
=

m=2

i = 0.04
t = 4 years
n=8

Ai
1(1+i )
21,000 (0.04)
1(1.04)

= 3,119.08
b)
R
0

R
1

R
2

R
3

R
yrs

The remaining liability after the 5th payments is the present value of the
3,119.09
remaining periodic payments.
3,119.09
3,119.09

1(1+i)

A=R
i
3

1(1.04)

A = 3,119.08
0.04
= 8,655.73

c) Amortization schedule
Period
1.
2.
3.

Balance
Payment
21, 000.00 3,119.08
18, 720.92 3,119.08
16, 350.68 3,119.08

Payment for
Interest Paid
Principal
840.00
2,279.08
748.84
2,370.24
654.03
2, 465.05

4.
13, 885.63 3, 119.08
5.
11, 321.98 3, 119.08
6.
8, 655.78 3, 119.08
7.
5, 882.93 3, 119.08
8.
2, 999.16 3, 119.08
Total
24, 952.64
The actual value is 20, 999.94. This is due to

555.43
452.88
346.23
235.31
119.97
3, 953.10
rounding error.

2,563.65
2, 666.20
2, 772.85
2, 883.76
2, 999.11
21, 000

Exercise 4.1
1. A debt of 70, 000.00 with 8% converted quarterly must be paid at the end
of every 3 months for 3 years.
a) Find the size of each payment.
b) Find the remaining liability after the 4th payment.
c) Construct the amortization schedule.
2. Nikko borrows a certain sum with interest at 9% compounded semi-annually.
The obligation will be discharged by paying 7, 000 every end of 6 months for
4 years.
a) What is the present value of Nikko's debt?
b) How much of his 3rd payment is interest and how much goes to
repayment of principal?
c) Prepare the amortization table.
3. A loan of 100, 000 with interest at 10% converted quarterly will be settled
by paying 12,000 at the end of every 3 months.
a) How many payments amounting 12, 000 will be made?
b) What final smaller payment is needed?
c) Construct the amortization schedule.
4. To settle a certain amount borrowed at 11% compounded semi-annually,
Alina pays Andro 15, 000 every semi-annual period for 2 years.
a) Determine Alina's loan.
b) How much of her 3rd payment is interest and how much is payment of
principal?
c) Construct the amortization schedule.
5. An item worth 33, 700 is purchased for a down payment of 5, 000. Interest
is computed at 6% converted monthly and the balance to be settled with
payment of 1, 500 at the end of each month.
a) Determine the number of 1, 500-payment needed.
b) Find the size of final payment.
c) Construct the amortization schedule.
6. A loan of 14, 000 bearing a 15% interest converted annually is to be
amortized with equal yearly payments for 10 years.
a) Find the size of each payment.
b) What should the outstanding liability be just after the 4th payment?
c) Construct the amortization schedule.

7. Aster borrows 360, 000. She plans to amortize her debt with equal quarterly
payments for 2 years. Interest is allowed at 11% converted quarterly.
a) Find the quarterly cost of his debt.
b) By how much is the debt reduced by the 3rd payment?
c) Construct the amortization schedule.
8. The cash value of motorcycle is 32, 500. Andro brought the item by
installment. He pays 7,000 down payment and promised to settle the balance
with equal payments due at the end of every month for 1 year. Interest is
converted monthly at 12%
a) How large should each installment payment be?
b) After 1 year, how much of his balanced is reduced?
c) Construct the amortization schedule.
9. Prepare the amortization table for the extinction of a debt of 12,000 with
interest at 8% compounded bi-monthly in equal bi-monthly installment for 1
year.
10. Construct the amortization schedule of an obligation amounting 145,000
bearing 18% interest compounded semi-annually for 5 years, to be paid in
equal semi-annual payments.
4.2 Sinking Fund
Sinking fund refers to a fund created by making periodic deposits to
anticipate the need of paying a large amount of money at some future dates.
Geometric progression formulas for ordinary annuity;
S=

R [ ( 1+i ) 1 ]
i

R=

S (i)
1(1+i)

Illustrative Examples
1. A fund is created by making equal monthly deposits of 3,000 at 9%
converted monthly.
a. Determine the sum after half year.
b. What is the amount in the fund after the 4th deposit?
c. Construct the sinking fund schedule for a 6-month period.
Given: R = 3,000

j = 9% or .09

Solution:
a) S=3,000

[ (1.0075 )6 1 ]
.0075

m = 12

S=3,000(6.113631847)
S=18, 340.89
b) n = 4
S=3,000

[ (1.0075) 1 ]
.0075

S=12,135.68

c) Sinking fund schedule


No. of
payme
nts
1
2
3
4
5
6

Perio
dic
Depo
sit
3,00
0
3,000
3,000
3,000
3,000
3,000

Interest
in Fund

Increase
in Fund

Amount
in Fund

3,000.00

3,000.00

22.50
45.17
68.01
91.02
114.20

3,022.50
3,045.17
3,068.01
3,091.02
3,114.20

6,022.50
9,067.67
12,135.68
15,226.70
18,340.90

2. Three years from now, Mr. T needs 30,000 to liquidate a certain debt, at 6%
converted semi-annually.
a. How much must he deposit at the end of every 6 months to provide for the
payment of the debt?
b. Prepare a sinking fund table showing the growth of the fund for 3 years.
a Given:
S = 30,000
j = 6% = .06
m=4
R=
R=

30,000(.05)
[ (1.015)61 ]
450
.093443263

R = 4,815.75

b Sinking fund table


Payme
nts
1
2
3
4
5

Deposi
t
4,815.
76
4,815.7
6
4,815.7
6
4,815.7
6
4,815.7
6

Interest
0

Increase
in Fund
4,815.76

Amount
in Fund
4,815.76

72.24

4,888.00

9,703.76

145.56

4,961.32

14,665.08

219.98

5,035.74

19,700.82

295.51

5,111.27

24,812.09

4,815.7
6

372.18

5,187.94

30,000.03

Exercise 4.2
1 A fund is being created by paying 2,500 at the end of each year a 10.25%
compounded annually for 4 years.
a How much money is in the fund just after the 3rd deposit?
b Construct a sinking fund schedule for 4 years.
2 Mr. Imperial will deposit 10,000 at the QUAK Bank at the end of each
quarter for 2 years. If the banks gives out 9% compounded quarterly, find
the amount to his credit just after the last deposit.
3 What monthly payment into a sinking fund at 8% compounded semi-annually
will be needed to raise 50,200 at the end of 2 years and 6 months?
4 A fund of 50,500 is to be raised by 6 semi-annul payments at 6% converted
every 6 months.
a Find the semi-annual payment
b Construct a sinking fund table.
5 Prepare a sinking fund table that shows the growth of fund in 5 months if
1,300 is to be paid at the rate of 12% converted monthly.
3 Sinking Fund Method of Retiring a Debt
A typical loan of moderate size to an individual or a corporation is discharge by
the amortization method.
Illustrative Example
1 The principal of a loan 40,000 will be at the end of 15 years by the
accumulation of a sinking fund by quarterly deposits, and interest will be
payable on the debt quarterly at the rate of 9%.
a Find the quarterly expense of the loan to debtor if his sinking fund is
invested at 8% compounded quarterly.
b Find the book value of the debt just after the 4th deposit.
c Find the quarterly expense of the debt if he discharge all the liability as to
principal and interest by paying his creditor equal sums at the end of each
3 months for 2 years.
Given:

S = 45,000
n=6

Solution:
R=

S (i)
[ (1+i)n 1 ]

R=

40,000(.02)
[ (1.02)n6 1 ]

R = 6,341.03

The interest payable quarterly to the creditor is 40,000(.0225) = 900.


Therefore, the quarterly expense of the debt is 6,341.03 + 900 = 7

Chapter 5
Depreciation, Capitalization and Perpetuities
Objectives: At the end of the lesson students should be able to:
a. calculate the rate of depreciation, periodic payments using different methods
b. participate actively in class discussion
5.1 Types of depreciation
1. Normal Depreciation
a. Physical Depreciation is due to the lessening of the ability of a property to
produce a result
b. Functional Depreciationis due to the lessening in the demand for the function,
which the
property was designed to render.
2. Depreciation Due to Changes in Price Levels
3. Depletionrefers to the decrease in the value of a property due to the gradual
extraction of its
contents.
Formula: W=PL ; B = P E
where: W=wearing value B = book value
P=price
E = accrued value
L=salvage value
5.2 Three Most Common Types of Depreciation
1. Straight-line Method
Denote:
P= original cost
L= scrap value/resale/salvage value
N= number of year/s (estimated life)
D= annual depreciation
d= 1/n
E=accrued value (tD)
B= book value
Formulas:
D=

PL
n

or D= d (W)

E= tD;
B=P-E or B=P- tD
In Straight line method, the total depreciation over the given life of a property is evenly
distributed, and this yearly depreciation is congruent to each other during the propertys
estimated life or period in n years.
Thus, d= d1=d2=d3=dn=D/n
The total depreciation D after a certain period of time or at the end of n years can be
obtained by the given formula:
D n=n (d)
Where:
D= total depreciation of the property over a period of time`
n= number of years (estimated)
d= annual depreciation
A book value after a particular year
Formula:
B= P-n (d)

Where:
B= book value
P= original cost
D= annual depreciation
N= number of year
Example: A motorcycle cost is 80, 000.Its depreciation value is 5,000 annually.
Find:
a. total depreciation at the end of 8 years
b. the book value at the end of 12 years
Given:
P=80,000
D=5,000
Required: Dn=?
B=?
Solution:
a. Dn= n (d)
b. B= P-n (d)
=8(5, 000)
=80,000 12(5, 000)
=40, 000
=80, 000- 60,000
=20, 000
2. Sum of the Years Method
Let K

n ( n+ 1 )
2

Where k=sum of the years digit or k=1+2+3+4+(n-1)+n


With k taken as the denominator and the terms of K taken in the reverse order as
numerator, then the following fractions:

n1

d1= k , d 2 k

,,dn-1 = k

, dn= k

Where dn is the rate depreciation to be applied to the wearing value, to obtain the
depreciation charge Dn=dnW since the sum of the numerations;

d1+d2+dn K =1

then,

D1+D2+ Dn=(d1+d2+dn)=W

Example: An equipment costing 4,500 will have a salvage value of 3,000 when retired at
the end of 5 years. Solve the total amount of depreciation every year.
Given: P=4,500; L=3,000;
n=5
Solution: Since n=5 then, k
K
=

n ( n+ 1 )
2

n ( n+ 1 )
2

5 ( 5+1 )
2

30
2

=15
Alternative solution: Since n=5 then, k=1+2+3+4+5=15
Solve for the wearing value W.
W =P-L
=4,500 - 3,000
5
= 15
=1,500
d1

andn=5

, where k=15

=.333333
D =d1 (W)
s =.333333 (1,500

5 (6 )
2

=500
d2 =

n1
15

51
15

=.2
D = d3 (W)
=.2 (1,500)
=300

=.2666666
D=d2 (W)
=.2666666(1,500)

d4 =

n3
15

53
15

=400
d3=

n2
15

52
15

=200
d5 =
=

n4
15

( 54 )
15

=.0333333
D = d5 (W)

=.1333333
D = d4 (W)

=.0333333(1,500)
=100

=.1333333(1,500)

3. Sinking Fund Method


1. W= r
where R replacement deposit

S n

depreciation charge for any year = increase in the depreciation fund at the end of the
year.
the amount of fund after the (k-1)th is:
2.
Rsk-1 i
the depreciation charge at the end of the kth year.
3.
Dk = Rsk i Rsk-1 i
Example:
A machine which costs 20,000 will have a scrap value of 2,000 when worn out at
the end of 5 years. Under the sinking fund method at 5%, find the annual charges for
depreciation. Prepare a schedule of depreciation.
Given: P = 20,000L = 2,000
Solution: a. W = PL
=20,000 2,000
=18,000
b. R = W

1
[ ( 1i ) 1 ]
r

Where: R=replacement deposit

n = conversion period
r =rate
(By using geometric progression)

R = 18,000

1
( 1.05 )1
.05

= 3,257.55

A. Depreciation schedule
At the end of
the year

Interest due
in Fund (I)

Replacement
Deposit (R)

Depreciation
Charge of Inc.

Accrued
Depreciation

Book Value B)

(D)

0
1
2
3
4
5

0
0.00
162.88
333.90
513.47
702.02

0
3,257.55
3,257.55
3,257.55
3,257.55
3,257.55

0
3,257.55
3,420.43
3,591.45
3,771.02
3,959.57

(E)
0
3,257.55
6,677.98
10,269.43
14,040.45
18,000.02

20,000.00
16,742.45
13,332.02
9,730.57
5,959.55
1,999.98

B. Find the depreciation charge at the end of 4 years and five years without using the
schedule.
Solution:
E=Rsk i
=3,257.55s4 .05
Using Geometric Progression

a. E=R

( 1+i )1
r

=3,257.55

( 1+.05 )1
.05

b. E=R

=14,040.45 (approximation)

( 1+i )1
r

=3,257.55

]
( 1+.05 )1
.05

=18,000.02 (approximation)

5.3 Depreciation by a Constant Percentage of Declining Book value


Depreciation charge for any year = (book value/ the beg of the year) d

d= n
d=

2
n

; (natural rate of depreciation)


(Federal revenue act of 1954)

Example:
1. By the method using a fixed percentage of declining book value with rate equal or
twice the natural rate, find the depreciation charges and book value during the life
of 4,000 with life of 3 years.
Solution:
The natural rate of depreciation is

Hence d = n , d =

2
3

1
n

which is =

1
3

= .66666 or 67 %

D = .67 (4, 000) the depreciation charge at the end of first year.
D = 2,680
The leaving book value:
4,000 2,680 = 1,320
5.3.1 Constant Percentage Method
The resale value of an asset decreases most in the first few years.
the difference between the book value and the actual value may be partially
corrected by the constant percentage method in which the depreciation charges are
greatest in the early life of an asset and decreases in each succeeding period.
Example:
2. If the rate depreciation of a machine costing 20, 000 is 10% a year, set up a
schedule of depreciation for 5 years.
Solution:
Year
Beg of year
Charge
End of year
1
20,000
2,000
.00
18,000.00
2
48,000
1,800.00
16,200.00
3
16,200
1,620.00
14,580.00
4
14,580
1,458.00
13,122.00
5
13,122
1,312.20
11,809.80
Exercise 5.1
1. An article costing 10,000 depreciates to 600 in 12 years. What is the annual
depreciation?
Given:
solution:
P=10,000

a. W=P-L

b. d= n

L=600
n=12 years

=10,000600

=9,400

9,400
12

=783.53

2. A sewing machine that costs 20,000 has an estimated life of 10 years and has a
salvage value at 500. Compute for annual depreciation charge and rate of depreciation.
Given:
solution:
P=20,000

W=PL

d=

19,500
10

L=500
=20,000500 =1,950
n=10 years
=19,500
3. A bicycle depreciates from original costs of 8,000 to a salvage value of 400 in 6
years. What is the annual depreciation? Construct a schedule of depreciation.
Given:
solution:
P=8,000

a. W=PL

b. d

7,600
6

L=400
=8,000400
=1,266.67
n=6 years
=7,600
Depreciation schedule
Accrued depreciation (D n)
Yearly depreciation (d
Book Value (B)
Year(n)
n)
0
0
0
8,000
1
1,266.67
1,266.67
6,733.33
2
1,266.67
2,533.34
5,466.66
3
1,266.67
3,800.01
4,199.99
4
1,266.67
5,066.68
2,933.32
5
1,266.67
6,333.35
1,666.65
6
1,266.67
7,600.02
399.98
Exercise 5.1
1. An article costing 10,000 depreciates to 600 in 12 years. What is the annual
depreciation?
2. A sewing machine that costs 20,000 has an estimated life of 10 years and has a
salvage value of 500. Compute for the annual depreciation charge and the rate of
depreciation.
3. A bicycle depreciates from the original cost of 8,000 to a salvage value of 400 in 6
years. What is the annual depreciation? Construct a schedule of depreciation.
5.4 Perpetuity
considered a form of annuity, considering a Perpetuity of R payable at the end of
each period and let I be the first interest rate per conversation period. Let A be the present
value of the perpetuity. Apparently if A is invested at the rate I, the interest alone on this
fund must provide R at the end of each interest period, so that the capital of the fund may
remain and permit the payments to continue unchanged forever.
Formula No 1:

A=

R
i

j
m

1=

(Present Value of Simple Ordinary Perpetuity)

Example 1
If money is worth 6% compounded quarterly, find the present value
a. Of a perpetuity of 100 payable quarterly
b. Of an annuity of 100 payable quarterly for 20 years
Solution:
Given:
J = 6%
m=4
R = 100
t = 20 years
a. A =

R
i

100
.015

=6,666.67
b. To solve the annuity use Ran i
Where an i = 1(1+ I )-n
i
K = 100 a80 .015
K = 100

1(1+ I )
.015

5.5 Perpetuity at the End of Each K Interest Periods


Consider a perpetuity of W payable at the end of each k interest periods, where k
is an integer. Let A the present value of this perpetuity. Before finding A, let us ask what
payment R at the end of each period could equitably replace W at the end of k periods.
The amount of annuity of R per period for K period should equal W hence Rs k I = W or
Formula:
R = __W__
s k I
A = __W__
s k I

from A =

R
i

substitute (1) to the equation

(present value of general perpetuity)

Example:
1. Find the present value of perpetuity of 200 payable semi-annually if money is worth
4% compounded quarterly.
Solution:
A = _R_. __1__
i
s ki
A = _200_. __1__
s 2.01
1.1
A = (20,000) (.49751437)
A = 9,950.58
2. To maintain a car in good condition, 10,000 will be needed at the end of each year and
annually thereafter. What is the present value of all future maintenance at a rate of 10%
compounded semi -annually?
Solution:
A = _R_. __1__
i
s ki

K=
i=

12months
6 moonths
.10
2

=2

= .05

A = _10,000_. __1__
.05
s 2.05
A = 200,000 (.487804878)
A = 97,560.98 (round to the nearest centavo)
Exercise 5.2
1. If money is worth 6 % compounded quarterly, find the present value
a. of a perpetuity of 50 payable quarterly
b. of an annuity of 50 payable semi annually for 30 years.
2. How much payment at the end of each year forever can be provided by endowment of
1,000 invested at 7%?
3. If money is worth 6%, obtain the present value of a perpetuity of 10,000 payable
annually, with the first payment due at the end of 5 years.
5.6 Capitalization
It is the process of finding the present value of all future earnings.
Capitalized value of an enterprise is the present value of all earnings similarly to speak
of capitalizing any sequence of equal payments due in the future means to find present
value.
Capitalized cost It is the sum of the original cost and the present value of all
replacement costs which are assumed to continue forever.
Formulas:
2. Replacement cost R per period that could equitably replace the amount W at
the end of K periods:
R = _1__
s k i
3. Capitalized value (assumed to continue forever) = pr. Value of the perpetual
income.
A = _R_, when future earnings from ordinary perpetuity
i
A = _R_. __1__, when future earnings from general perpetuity
i s ki
4. Capitalized cost = Original cost + present value of infinitely many future
replacements
C=

P+

R
i

, when replacement cost per period from an ordinary perpetuity

5. Capital (C) as present value of the perpetuity of W payable at the end of K


periods
C=

P+ _R_. __1__, when replacement costs per period from general

perpetuity
i

s k i

Example:
1. It is estimated that the maintenance of a certain section of a LRT railroad will
required 2,000 per kilometer at the end of every 4 years. If money is worth
10%, find the capitalized cost of the maintenance per kilometer.
Solution:
Replacement cost per period:
R =__W_ =_2,000_
s k I
s 4 10%
=

2,000
4.641

= 430.9416074
Capitalized cost per kilometer:
C=

R
I

430.9416074
0.1

C = 4,309.42
C = __W__ =__2,000__
is k i
0.1(4.641)
C = 4,309.42
Formula:
Annual investment cost:
M = Pi +__W_ , W = P L
S n i
P = Original cost
W = Wearing value
L = Scrap value
Capitalized costs Based Annual Investment Cost of the Asset
Example:
2. Find annual investment cost (M) and also the capitalized cost (K) of the machine
by first finding the value of K and then using M = Ki. Money worth 4% machine
will cost 15,000 = life 8 years
Final salvage value 3,000
Solution:
Method 1
W=PL
P = 15,000
L =3,000
W = 15,000 3,000
W = 12,000
K = P + =__W__
is k i
K = 15,000 + ___12,000_____
(0.04)(s8 0.04)
K = 15,000 + 32,558.34961
K = 47,558.35
M = Ki, Annual investment cost
M = (47,558.35) (0.04)

M = 1,902.33
Method 2
M = Pi + __W , W = P L
sni
M = (15,000)(0.04) + _12,000_
s8 0.04
M = 6,000 + 1,302.33
M = 1,902.33
Annual investment cost
At M = Ki
K=

M
I

1,902.33
0.04

K = 47,558
Exercise 5.3
1. It is estimated that a certain mine will yield a net profit of 200,000 at the end of each
year practically forever. Approximately, what is the capitalized value of this mine if money
is worth 7%?
2. How much is the capitalized value of an asset of 2,500 payable at the end of each
month, indefinitely, if money is worth 6% (m = 12)
3. At 5 % find the capitalized cost of an asset whose cost is 50,000, life is 10 years, and
final salvage value is 5,000.

3.1 Basic concepts and Terminologies


An annuity is a sequence of payments, usually equal, made at regular
intervals. Some examples of annuities are quarterly payments of a lots
amortizations, premiums on insurance, periodic pensions, monthly payments on
installment purchases, among others.
Annuities are divided into: annuity certain and contingent annuity or annuity
uncertain. Annuity certain; when the term of the annuity is fixed. A contingent
annuity is one for which the first and last payments or both cannot be foretold
accurately.
The following symbols will be used in dealing with ordinary annuity formulas:
R - periodic payment of the annuity
n - total number of payments
i - interest per conversion period
S - amount of an annuity
A - present value of an annuity
3.2 Amount and Present Value of an Ordinary Annuity
The amount or final value, denoted by S, of an ordinary annuity is the sum of
all the accumulated value of the set of payments due at the end of the term,
while the present value of an annuity, denoted by A, is the sum of all the
discounted value of several payments due at the beginning of the term.

A and S are related by the equations


and
A = S(1
-n
+ i)
S = A(1
where + i)n A is the present value of S due in n periods and
S is the amount of A for n periods

Illustrative Example
Consider an ordinary annuity of 10,000 per year payable for 3 years with
money worth 10%.
To find the amount S of the annuity, add the accumulated payments of
each period to the end of 3 years.
Term

10,000 =P10,000
10,000(1 +.10)1 = 11,000
10,000(1 +.10)2
= 12,100
S
= P33,000
To find the present value of an annuity, add theTerm
discounted
payments.

10,000(1 + .10)-1 = 9,090.91


10,000(1 +.10)-2 = 8,264.46
10,000(1 +.10)-3
=
7,513.15
A
=
24,868.52
Hence, from the relation

A = S(1 + i)-n
and S = A(1 + i)n
A = 33,100(1 + .10)-3
and S = 24,868.52(1 + .10)3
A = 24,868.52
S = 33,100

3.3 Ordinary Annuity Formulas


The periodic of R for n periods.
Term
R
0

R
n-1
R
R(1
.
.
.
R(1
R(1
R(1

(periods)

+ i)1

+ I)n-3
+ I)n-2
+ I)n-1

S = sum of the accumulated values of R at the end of the term


(1) S = R + R(1 + i) + . . . R(1 + i) n-3 + R(1 + i)n-2 + R(1 + i)n-1
multiplying (1) by (1+i)
(2) (1 + i)S = R(1 + i) + R(1 + i)2 . . . R(1 + i)n-3 + R(1 + i)n-1 + R(1 + i)n
subtracting (1) from (2) we get
(1 + i)S S = R(1 + i)n R
iS = R[(1 + i)n 1]

R (1+ i)n1
S
S=
=R
1
n
(1+i) 1
or
i

if we letSn i =

n
(1+i) 1
the formula for S can be written as
i

where

Sn i =
Rsn i

S id the amount of an ordinary annuity


R is the periodic payment
j
I = m = interest rate per period
N = mt = number of payments

The symbol, Sn I, is read as S angle n at I


Sn i =
n
(1+i)
1
To derive
the
present value of an ordinary annuity we use the relation
i + i)-n
A = S(1

A = RSn i(1 + i)-n


A= R

[
[

( 1+i ) 1
(1 +
i
n
1(1+i)

=
n
i
if we let
i
n
1(1+i )
then the formula
i of A can be written as
A= R

A=
Ran i

where A is the present value of the annuity an I, is read as a angle n at i


an i =

1(1+i)n
i
Illustrative Example
1
Find the amount and present value of an annuity of 1,500 payable for
2 years if money is worth 105 compounded semi-annually.
j
.10
Given
R = P1,500
i = m = 2 =.05
n = mt = 2(2) = 4
periods

j = 10% - .10
i = 2 years
m=2
To find S
S
S
S
S

or

=
=
=
=

Rsn i
1,500svn .05
1,500(4.310125)
P6,465.19

S=R

( 1+.05)4 1
i

s = 1,500

]
4

( 1+.05) 1
.05

s = 1,500(4.310125)
s = 6,465.45

to find A
A
A
A
A

=
=
=
=

Ran i
1,5004 .05
1,500(3.54595)
P5,318.93

or by using the relation


A = S(1 + i)-n
A = 6,465.19(1 + .05)-4
A = P5,318.93

Illustrative Example 2
Mrs. Cruz purchased a house and lot. If she paid 400,000 as down
payment and promised to pay P5,000 every 3 months for the next 10 years
at 15% compounded quarterly, find the cash value of the house.
Given:

DP = 400,000
R = 5,000
m=4

j = 15%

t = 10 yrs.

Solution:
The down payment is not part of the annuity. The cash value of the
house is 400,000 plus the present value of the ordinary annuity of 40
monthly payments of P5,000 each. The cash value is
Cash Value = down payment + Ra n I
= 400,000 + 5,000 40 .0375
= 400,000 + 5,000(20.5509999)
= 400,000 + 102,754.95
Cash Value = 502,754.94

3.4 Periodic Payment of an Ordinary Annuity


a.) Periodic payments of S
S
R=
s n i or R =
b.) Periodic payment
of A
A
R=
an i or R =
Illustrative Example 1
In order to have 300,000 at the end of 15 years, how much must be
deposited in a fund every 3 months if money Is worth 8% compounded
quarterly?
Solution:
Given:
S = 300,000
t = 15 years
n = 60 periods
j
8
i = m = 4 = .02

Find R:

R=

s n i

R=

300,000
s 50 .02

R=

300,000
114,05154

R = 2,630.38

Illustrative Example 2
Mr. Tuy bought a refrigerator that cost 19,500. He paid 6,000 as down
payment and the balance will ne paid in 36 equal monthly payments. Find the
monthly payment if money is 15% compounded monthly.
Solution:
A = Cash Value Down
A = 19,500 6,000 = 13,500
n = 36
i=

.15
12

Find R:

= .0125

R=

A
an i

R=

13,500
1( 1+.0125 )36
.0125

R=

13,500
28.84726737

13,500
a.36.0125

Exercise 3.1
1. Analyn bought a dining set. She paid 2,500 as down payment and promised
to pay 800 at the end of each month for one year. What is the cash
equivalent of the set if the interest rate is 10% compounded monthly?

2. Find the amount present value of an annuity of 5,240 payable at the end of
1
every 6 months for 5 years and m6 months at 5 2 converted semiannually.
3. What is the present value of 1,230 due at the end of every three months for
3
3 years and 6 months, if money is worth 9 4 % compounded quarterly?

4. An item can be bought for 25,300 as down payment and 12 semi-annual


payments of 480 each. What is the cash equivalent of the item if money is
1
worth 5 2 compounded semi-annually.

5. If Cherry will deposit P1,500 every end of each quarterly in 9 years at the
1
rate of 6 2 compounded quarterly, how much will be the amount of her
savings at the end of 5 years?

6. What is the cash value of a set of Science Encyclopedia that can be


purchased for 12,500 as down payment and 1,450 at the end of each
month for 18 months at 15% compounded monthly?
7. In a series of monthly payments of 1,580 for 10 years, what is the cash
value of an annuity if money is worth 12% compounded monthly?
8. A man deposits 12,500 at the end of every six months in a bank which
credits interest at 9% compounded semi-annually. How much will he have
after 19 years?
9. Mr. Chua purchased a car. If he paid 400,000 as down payment and
1
promised to pay 10,500 every month for 5 years at 7 2 compounded
monthly, what is the cash value of the car?
10.

Find the present value if an annuity whose semi-annual payment is


3
2,500 payable for 5 years at 8 4 compounded semi-annually.

11.
A man purchased a DVD player worth 8,500. He paid P2,000 cash and
agreed to make 15 monthly payments. If interest is 18% (m=12), find the
monthly payment.
12.
Marga borrowed P250,00 and agreed to repay her obligation by making
equal quarterly at the end of every months for 4 years. What is the periodic
payment, if money is worth 12.5% compounded quarterly?

13.

1
How much should be invested each year in a fund paying 10 2

effective to accumulate to P250,000 in 5 years?


14.

Find the monthly payments for 30 periods to discharge an obligation of


3
300,000, if money is worth 9 4 compounded monthly.

15.

1
How much should one deposit monthly in a fund that earns 12 4

compounded monthly in order to have 350,000 in 3 years and 6 months?

3.5 Finding the Interest Rate of an Ordinary Annuity


These formulas are:
nR
1
a) (n2 - 1)i2 6(n - 1)i + 12
=0
S

b) and (n2 - 1)i2 + 6(n + 1)i + 12

)
(1 nRS )

=0

Example 1
A refrigerator can be purchased for 9,000 cash down and P630 a month for
18 months. Find the interest rate charged if it is compounded monthly.
Solution:

A = 9,00 900 8,100


R = 630
n = 18
m = 12

Formula:
Since A is known

(1 nRA ) = 0
18(630)
(18 - 1)i + 6(n + 1)i + 12 ( 1 8,100 )
(n2 - 1)i2 + 6(n + 1)i + 12
2

=0

323 i2 + 114i 4.8 = 0

To find i, we use the quadratic formula for ai2 + bi + c = 0, then


b b24 ac
i=
2a

(114 )24 (323)(4.8)


i = -114
2( 323)
i=

114 138.555404
646

i = -.3909526 or .03801146

Here we get two values of i, we disregard the negative value of i. Therefore,


the periodic interest rate is
i
i
i
i

=
=
=
=

3.80% and the nominal interest rate charged is


ixm
(.03801146)12%
45.61%

Example 2:
Payments of 750 each are made every 6 months in 3 years. At what rate
compounded semi-annually will these payments amount to 5,100?
Solution:

S = 5,100
R = 750
n = mt = 2(3) = 6

Formula: Since S is known


(n2 - 1)i2 6(n - 1)i + 12
(62 - 1)i2 6(6 - 1)i + 12

(1 nRS ) = 0
(1 6(750)
5,100 ) = 0

35i2 30i + 1.4117647 = 0

To find i, we use the quadratic formula. We have


30 3024 (35)(1.4117647)
i=
2(35)
here we set two values of i
i = .887170603 or.0494722647

Since i = .049972253 (2)


j = im = .049972253

Therefore, the interest rate charged is 9.99944 compounded semiannually.


Exercise 3.2
1. in purchasing a washing machine worth P21,000, a buyer pays 3,500 cash
and agrees to make 12 monthly payments. If the monthly payment is 1,580,
find the interest rate m charge.
2. Deposit of 8,380 are made every end of six months. At what rate
compounded semi-annually will these deposits amount to P112,400 in 14
years?

3. A television set costs P10,440. It is purchased by a down payment of 4,440


and monthly payments of P 382.84 for one year and 6 months. Find the rate
of interest converted monthly. Use either interpolation or the given
derivation of i.
4. A man invests 9,375 every six months. If he has 200,000 in five years, at
what nominal rate compounded semi-annually did hi investment earn?
3.4 The Term of an Ordinary Annuity
The number of payments can be determined when the amount or present
value, the periodic payment and interest rate are given
Formula in finding the term, n is
n=

Ac
Example 1log 1
R 15,000 with interest at 15% compounded quarterly. She
Victoria borrows
log the
(1+1)
will discharge
debt by paying P950 quarterly.
a) How many payments of 950 are required?
b) How much would the final payment be if it is made the day after the last
P950 payment?

c) How much would the final payment be if it is made 3 months after the last P950
payment?

Solution:
Given: A = 15,000
R = 950
i = .0375
a) Substitute the given values into the formula
A
log 1 c
R
n=
log ( 1+ i)

n=

n=
n=

15,000(.0375)
950
log ( 1+.0375)

log 1

log.407894737
log ( 1.0375)
.389451898
.015988105

n = 24.35885291

This means that there will be 25 quarterly payments of 950 and a


final irregular payment.
b) To find the final payment, the 24th payment in this case.

Let x be the irregular payment to be paid on the 24th payment. Make a


time diagram using the present as the focal date.
Ordinary annuity of 24 months
0950950950 950950

. . .

1 2 3 23 24

Focal date

x(1.0375)-24 + 950 a24 .0375

= 15,000

x(.413319095) + 14,862.58 = 15,000


x = P332.47

Therefore the final 24th payment is

= 24th payment + final irregular payment


= 950 + 332.47
= P1,282.47

c) To find the concluding payment (irregular payment on the 25th payment)


Let y be the final payment on the 25th period
950 950 950
0

.3 . .

950 950 950 y


22

23 24 25

ordinary annuity of 24 payments

Using the present as the focal date


y(1.0375)-25 + 950 an .0375 = 15,000
y(.39837985) + 14,862.58 = 15,000
x = 344.85

Exercise 3.3
1. Lulu borrows 240.000 with interest at 12% compounded monthly. She will
pay the debt by paying 5,300 monthly
a. Find the number of regular payments to be made.
b. How much will be the final payment if it is made one month after the last
5,300 payment?
2. The present value of an annuity is 270,000 whose semi-annual periodic
payment of 15,600 will be needed?
3. Margarita borrowed 500,000 and agreed to repay it in equal quarterly
installment of 18,500 with interest at 10.5% compounded quarterly. If the
first payment is due in three months after borrowing the money.
a. How many full payments must the debtor make
Ordinary annuity of 24 months
0950950950 950950
1 2 3 23 24

. . .

b. How much would the final payment he if it is made on the day of the last
18,500 payment?
4. A car is worth 730,000. The buyer paid 350,000 down payment and
3
promised to pay 7,660 every month, if he is charged 7 4 compounded
monthly,
a. How many full payments must be made?
b. How much will be the final payment?
5. A fund of 400,000 is to be created by making a quarterly deposit of
15,000. If the fund earns 12% compounded quarterly,
a. How much deposits of 15,000 are required?
b. How much would the final deposit be if it is made three months after the
last 15,ooo deposit?
3.4 Annuity Due
An annuity due is one for which the first payment occurs immediately. It is
also defined as an annuity in which payments are made at the beginning of
each period.
The following symbols re used in dealing with annuity due formulas:
R = periodic payment of the annuity due
n = total number of payments
i = interest per conversion period
S = amount of an annuity due
A = present value of an annuity due
3.7.1 Present Value of an Annuity Due
The present value of an annuity due, which is denoted by A, is the sum of the
present values of the payments.
Term
0
1
R
R

3
R

...

R ...

n-1 n
R

The formula in finding the present value of an annuity due is:


A = (1st payment) + (present value of remaining payments)
A =A +
Ran1 i
Example 1
Marlon purchased a car. He paid 150,000 down payment and 10,000
payable at the beginning of each month for 5 years. If money is worth 12%
compounded monthly, what is the equivalent cash price of the car?

Given:

R = 10,000

j = 12%

12
12

Down payment = 150,000 i =


m = 12
t = 5 years
n = 60

1%

A = ? Cash Equivalent - ?

Ran1 i

A =R+

A = 10,500 +

10,500601 1

A = 10,500 + 10,500 a59 1


A = 10,500 + 10,500

59

1(1.01)
.01

A = 10,500 + 466,248.18
A = 476,748,18 (Total Installment Payment)
Cash Equivalent = Down Payment + Total Installment Payments
Cash Equivalent = 150,000 + 476,748.18
Cash Equivalent = 626,748.18

3.7.2 Amount of an Annuity Due


The amount of an annuity due, which is denoted by S, is the sum of the
accumulated values of the payments at the end of the term.
0
1
R
R

3
R

...

n-1 n
R

R ...

The formula in finding the amount of an annuity due is:


S = {Value of an annuity of (n +1) payments on the last payment date} R
S = Rsn+1 iR
Example 1
If 500 is deposited in a bank at the beginning of each 3 months for 10
years and money is worth 8% compounded, how much is in the fund at the end
of 10 years?
Given:
R = P500

j = 8%

M=4

i=

t = 10 years
n = 40
S=

Rsn+1 iR

8
4

= 2%

S=?

S=

500 s40+ 1 2 500

S = 500

(1.02)411
.02

-500

S = 31,305.01 500
S = 30,805.01

Example 2
Emil invests 5,000 at the beginning of each six months. He makes his
first deposit on January 19, 2003. How much will be in his account on January
19, 2015, if money is worth 9% compounded semi-annually?
R = 5,000
m=2
t = January 19, 2003 January 19, 2015 = 12 yrs.
n = 24
j = 9%

9
2

i=

=4 2 %

S=?

Rsn+1 iR

S=
S=
S=
S=

5,000 s

5,000 s

5,000

24+1 4

25 4

1
2

1
2

5,000

5,000
13

(1.045) 1
.045

-5,000

S = 22,826.05 5,000
S = 217,826.05

3.7.3 Periodic Payment R of an Annuity Due

The following formulas are sued in finding the periodic payment R of


an annuity due: A
S
R=

1+an 1 i

R=

Sn +1 i1

Where:
A = present value of an annuity due
S = amount of an annuity due
R = periodic payment of the annuity due
i = interest per conversion period
n = total number of payments

Example 1
What equal deposits should be placed in a fund at the beginning of each
year for 15 years in order to have 1,500,000 in the fund at the end of 15
years, if money accumulates 12%?
Given:
S = 1,500,000
m=1
t = 15
n = 15
R=

A
S n+1 i1

R=

P 1,500,000
S 15+1 12 1

R=

P 1,500,000
S16 12 1

R=

P 1,500,000
41.75328042

j = 12%
i = 12%
R=?

R = 35,925.32

Exercise 3.4
For each problem draw a diagram illustrating the data and the solution
1. If money is worth 16% compounded quarterly, find the present value and the
amount of annuity due of 1,500 payable quarterly for 10 years.
2. An investment of 5,000 is made at the beginning of each month for 8 years
and 7 months. If interest is 12% compounded monthly, how much will the
investment be worth at the end of the term?
3. At retirement, Mr. Nolram Ocampo finds his share f a pension fund is
2,500,000. What payment will this provide at the beginning of each month
for 25 years, for him or his estate, if the fund invested at 18% compounded
quarterly?
4. Mr. Jason Raymundo agrees to pay 2,000 at the beginning of each 6 months
1
for 5 years. If money is worth 12 2 % compounded semi-annually, find a)
the present value of his debt, before he makes a payment; b) Mr.
Raymundos outstanding liability just before his 7th payment; and c) his
outstanding liability just after the 7th payment.

5. A house and lot is bought for 1,000,000 down payment and 5,000 payable
at the beginning of each month for 5 years. What is the equivalent cash price
of the house and lot, if interest rate is 14% compounded monthly?
6. On January 19, 2003, Mr. Berlindo Santos opened a saving account for his
wife with an initial deposit of 10,000 in a bank paying 6% compounded
quarterly. If Mr. Santos continues to make quarterly deposits of the same
amount until July 19, 2008, how much will be in the account at the end of the
term? (Hint: Count the time by months.)
7. A machine will be replaced 10 years from now at a cost of 750,000. How
much should the owner save at the beginning of each quarter in order to
3
replace the old machine if his savings earn interest at 8 4 % compounded
quarterly?
8. What equal deposits should be placed in a fund at the beginning of each year
for 20 years in order to have 2,500,000 in the fund at the end of 20 years, if
1
the money accumulates 10 2 %?

9. Mayette agrees to make equal payments at the beginning of each 3 months


for 12 years, to pay all interest and principal in purchasing a lot worth
1
1,200,000 cash. If money is worth 12 2 % compounded quarterly, find the
quarterly payment.
10.
A loan of 200,000 will be settled by 25 equal payments, the first of
which is to be made immediately. Find the quarterly payment, if money is
worth 10% compounded quarterly.

3.8 Deferred Annuity

Is an annuity whose term does not begin until the expiration of a


specified time
Ad

Sd

...

...

2
3
d d1 d2
deferment period n periods
Ad
(d)
No payments

d+n-2

d+n-1 d+n

Payments are made

...

...

2
3
d d1 d2
deferment period n periods
(d)
No payments

Payments are made

Sd
d+n-2

d+n-1 d+n

The formula in finding the present value


Ad

Ad

of the deferred annuity is

= {present value of annuity with term (d + n) periods} {present


value of annuity with term d periods}
A d =Ran+d i Rad i

Where:
A d = present value of deferred annuity
d=

value of deferment

n=

total number of payments

R=

periodic payment of the deferred annuity

i=

interest per conversion period

Example 1
A sequence of quarterly payments of 3,500 each, with the first one due
at the end of 3 years and the last of the end of 10 years. Find the present
value of the deferred annuity, if money is worth 16% compounded quarterly.
Given:
R= 3,500

Solution:
16
i=
4

j = 16%

m=4

= 4%

d = 3 x 4 = 12; 12 1 = 11
n = 7 x 4 = 28; 26 + 1 = 29
40
11

29
RRRRRRRRRRRR

2 yrs

RRRRR

10

7
10

Ad

Ran+d iRad i

Ad

Ad3,500 a29 +114 3,500 a11 4

A d 0=
Ad

...

...

3,500
1
2 a 4034 3,500 a11 4d d1 d2
deferment 40
period n periods

(d)
1(1.04)
= 3,500 No payments
.04

Sd

d+n-2
11

1(1.04)
Payments
are made

- 3,500

.04

d+n-1 d+n

Ad

= 69,274.71-30,661.67

Ad

= 38,613.04

Example 2
Find the cash equivalent of an item that sells for 20,000 down payment
and 20 semi-annual payments of 5,500 each, the first is due at the end of 3
years, if money is worth 10% compounded semi-annually.
Given:

R = 5,500 Down payment = 20,000

Solution:
10
i=
2

j = 10%

m=2

= 5%

d = (3 x 2) 1 = 6 1 = 5
n = 20
Ad
25
5

20
R R R R R R R R R R R R R R R R R R R R R R

10

11

12

13

2 yrs

Ad

Ran+d iRad i

Ad

5,500 a20 +5 5 5,500 a5 5

Ad

5,500 a25 5 5,500 a 5 5

1(1.05)25
.05

Ad

= 5,500

Ad

= 77,516.70 23,812.12

Ad

= 73,704.58

- 5,500

1(1.05)5
.05

3.8.1 Periodic Payment R of Deferred Annuity


The formula in finding the periodic payment R of a deferred annuity is:
Ad
a d+ n ia d i
Example 1
Alex borrows 100,000 with interest at the rate 12% compounded semiannually. he agrees to discharge his obligation by paying a sequence of 10
R=

1
equal semi-annual payments, the first being due at the end of 5 2

years. Find

the semi-annual payment.


Given:
Ad

= 100,00

J
m
n
d

= 12%
=2
= 10
= 10

12
2

= 6%

Solution:
R=

Ad
ad +n i ad i

R=

P 100,000
a20+6 ia 10 6
P100,00

R=

20

1(1.06)
.06

][ ]

110
.06

R=

P 100,000
11,469921227.360087051

R=

P 100,000
4.109834169

R = 24,331.88

Exercise 3.5
For each problem, draw a line diagram illustrating the data and the solution
Find the present values of the deferred annuity which is described, at the
specific interest rate.
1. A farm costs 750,000 cash. Paolo will pay 150,000 cash and a sequence of
15 annual payments, the first due at the end of 5 years. If money is worth
12%, find the annual payment.
2. A house and lot is offered for sale 1,500,000 cash, or 500,000 cash and a
sequence of 20 semi-annual payments of 120,000 each, the first is due at
the end of 3 years, if money is worth 14% compounded semi-annually, find
the cash value of the house and lot.

3. Find the present value of a series of quantity payments of 2,500 each, the
first payment is due at the end of 4 years and 6 months, and the last at the
end of 10 years and 3 months, if money is worth 16% compounded quarterly.
4. Find the cash equivalent of a computer set that sells for 10,000 cash and a
sequence of 10 semi-annual payments of 3,500 each, the first is due at the
1
end of 3 years and 6 months, if money is worth 10 2 % compounded semiannually.
5. Jason borrows 50,000 and agrees to pay his obligation by making 15 equal
annual payments, the first is due at the end of 3 years . Find the annual
payment, if money is worth 12%.
6. Ryan obtains a loan of 250,000 with interest t 14% compounded quarterly.
He will discharge his debt by a sequence of equal quarterly payments, the
first is due at the end of 5 years and the last at the end of 10 years find the
periodic payment.
7. Find the present value of the pension of a man, now 60 years old, and who
will receive a pension of 10,000 per month for 15 years, with the first
payment to occur one month after his 65th birthday, if money is worth 18%
compounded monthly.
8. An investment in a stock market will yield no operating profit until the end of
5 years, when investor will receive P100,000 after that, he will receive
100,000 at the end of each quarter for 10 more years. Find the present
1
value of this income if money is worth12 2 % compounded quarterly,

9. On November 20, 2002 a house and lot was bought for 500,000 down and
12 quarterly payments of 50,000 each, the first is due on November 20,
2003. Find the cash value of the house and lot, if money is worth 15%
compounded quarterly.
10.
Aling Nelie won 25,000,000 in Mega Lotto. She invest winnings at
16% compounded semi-annually with the conditions that she receive 25
semi-annual withdrawals starting at the end of 2 years. Find the size of the
withdrawals.

3.8.2 Value of an Annuity in an Arbitrary Date


Example 1
A sequence of 5 semi-annual payments of 2,500 each will start with a
1
payment at the end of 2 2 years. If money is worth 12% compounded semiannually, find the sum of the values of these payment at the end of a)2 years,
1
b)4 2 years, c) 6 years, d)the actual present value of the payments.
Given;

R = 2, 500
j = 12%
m=2
i=

12
2

= 6%

n=5

Time Diagram:
R
0

2
3
4
1 yr.
2 yrs.

6
3 yrs.

R
7

R
8
4 yrs.

R
9

10
5 yrs.

11

12 13 14
6 yrs.
7 yrs.

Solution;
a) At the end of 2 years

b) At the end of

A = 2, 500 a5 6
A2 = 2, 500

1
2

years

S = 2,500 a5 6

1( 1.06 )
.06

S = 2,500

A = 10, 530.91

( 1.06 )51
.06

S = 14, 092.73

c) To find the value of the annuity at the end of 6 yrs, accumulate a) for 4 years, or

the result of b) for 1 2

years.

Value at the end of 6 years is = 2,500( a5 6 )(1.06)8


= 16, 784.67

Value at the end of 6 years is = 2,500( a5 6 )(1.06)3


= 16,794.67
d) To find the actual present value, discount the result of a) for 2 years or the

result of b) for 4 2

years.

Actual Present Value = 2,500 )(1.06)-4

or

=8,341.47
Actual Present Value = 2,500( a5 6 )(1.06)3
=8,341.47

Exercise
For each problem in the exercise, draw a time diagram showing the payment dates,
and the beginning and the end of the term of any ordinary annuity involved.
1. A sequence of 16 quarterly payments of 5,000 each will start with a payment at the

end of 5 2

years. If money is worth 18% compounded quarterly, find the sum of

the values of these payments at the end of a) 4 years b) 8 2

years c) 10 years d)

find the actual present value of the payments.


2. In starting a business venture, the owner is granted a loan of 750,000 at the

beginning of ach 6 months for 10 years. He agrees that money is worth 12 2 %


compounded semi-annually, and that he will discharge all accumulated liability by a
single payment at the end of 15 years. What amount must he pay to settle his
obligation?
3. If the fund accumulation at 15% compounded semi-annually, how much will be in the
fund at the end of 30 years if 5,000 is deposited in it at the end of each 6 months
for the first 25 years?
4. A fund of replacement of machinery in a plant must contain 300,000 at the end of
12 years. If the fund is invested at 10.75% compounded quarterly, what equal
deposits should be placed in the fund at the end of each 3 months just for the first 7
years?
5. The first payment of 10,000 by Paul is due in 2 years. Payment continues every 3
months until the 15th year. If money is worth 20% compounded quarterly, find the
annuity value at the beginning of the first year.

CHAPTER 4
Amortization and Sinking Fund
Amortization

Amortization is a means of repaying a debt by a series of equal time interval.


the periodic payments form an annuity in which the present value is the principal
of an interest-bearing debt. Hence we use the following annuity formulas;

A=R

R=

Where:

1(1+i)
i

Ai
1(1+i)

A = principal
R = periodic payment
i = interest per period
n = total number of payment periods

Examples:
1. An obligation of 21,000 with interest of 8% compounded semi-annually
must be paid at end of every 6 months for 4 years. a) Find the size of periodic
payment. b) Find the remaining liability just after making the 5th payment. c)
Prepare the amortization table.
Solution:

A = 21, 000
j = 8%

a) R =
=

m=2

i = 0.04
t = 4 years
n=8

Ai
1(1+i )
21,000 (0.04)
1(1.04)

= 3,119.08
b)
R
0

R
1

R
2

R
3

R
yrs

The remaining liability after the 5th payments is the present value of the
3,119.09
remaining periodic payments.
3,119.09
3,119.09

1(1+i)n

A=R
i

1(1.04)

A = 3,119.08
0.04
= 8,655.73

c) Amortization schedule
Payment for
Period
Balance
Payment
Interest Paid
Principal
1.
21, 000.00 3,119.08
840.00
2,279.08
2.
18, 720.92 3,119.08
748.84
2,370.24
3.
16, 350.68 3,119.08
654.03
2, 465.05
4.
13, 885.63 3, 119.08
555.43
2,563.65
5.
11, 321.98 3, 119.08
452.88
2, 666.20
6.
8, 655.78 3, 119.08
346.23
2, 772.85
7.
5, 882.93 3, 119.08
235.31
2, 883.76
8.
2, 999.16 3, 119.08
119.97
2, 999.11
Total
24, 952.64
3, 953.10
21, 000
The actual value is 20, 999.94. This is due to rounding error.

Exercise 4.1
1. A debt of 70, 000.00 with 8% converted quarterly must be paid at the end
of every 3 months for 3 years.
a) Find the size of each payment.
b) Find the remaining liability after the 4th payment.
c) Construct the amortization schedule.
2. Nikko borrows a certain sum with interest at 9% compounded semi-annually.
The obligation will be discharged by paying 7, 000 every end of 6 months for
4 years.
a) What is the present value of Nikko's debt?
b) How much of his 3rd payment is interest and how much goes to
repayment of principal?
c) Prepare the amortization table.
3. A loan of 100, 000 with interest at 10% converted quarterly will be settled
by paying 12,000 at the end of every 3 months.
a) How many payments amounting 12, 000 will be made?
b) What final smaller payment is needed?
c) Construct the amortization schedule.
4. To settle a certain amount borrowed at 11% compounded semi-annually,
Alina pays Andro 15, 000 every semi-annual period for 2 years.
a) Determine Alina's loan.
b) How much of her 3rd payment is interest and how much is payment of
principal?
c) Construct the amortization schedule.

5. An item worth 33, 700 is purchased for a down payment of 5, 000. Interest
is computed at 6% converted monthly and the balance to be settled with
payment of 1, 500 at the end of each month.
a) Determine the number of 1, 500-payment needed.
b) Find the size of final payment.
c) Construct the amortization schedule.
6. A loan of 14, 000 bearing a 15% interest converted annually is to be
amortized with equal yearly payments for 10 years.
a) Find the size of each payment.
b) What should the outstanding liability be just after the 4th payment?
c) Construct the amortization schedule.
7. Aster borrows 360, 000. She plans to amortize her debt with equal quarterly
payments for 2 years. Interest is allowed at 11% converted quarterly.
a) Find the quarterly cost of his debt.
b) By how much is the debt reduced by the 3rd payment?
c) Construct the amortization schedule.
8. The cash value of motorcycle is 32, 500. Andro brought the item by
installment. He pays 7,000 down payment and promised to settle the balance
with equal payments due at the end of every month for 1 year. Interest is
converted monthly at 12%
a) How large should each installment payment be?
b) After 1 year, how much of his balanced is reduced?
c) Construct the amortization schedule.
9. Prepare the amortization table for the extinction of a debt of 12,000 with
interest at 8% compounded bi-monthly in equal bi-monthly installment for 1
year.
10. Construct the amortization schedule of an obligation amounting 145,000
bearing 18% interest compounded semi-annually for 5 years, to be paid in
equal semi-annual payments.
4.2 Sinking Fund
Sinking fund refers to a fund created by making periodic deposits to
anticipate the need of paying a large amount of money at some future dates.
Geometric progression formulas for ordinary annuity;
S=

R [ ( 1+i ) 1 ]
i

S (i)
1(1+i)

R=

Illustrative Examples
1. A fund is created by making equal monthly deposits of 3,000 at 9%
converted monthly.
a. Determine the sum after half year.
b. What is the amount in the fund after the 4th deposit?
c. Construct the sinking fund schedule for a 6-month period.
Given: R = 3,000

j = 9% or .09

m = 12

Solution:
a) S=3,000

[ (1.0075 )6 1 ]
.0075

S=3,000(6.113631847)
S=18, 340.89
b) n = 4
S=3,000

[ (1.0075) 1 ]
.0075

S=12,135.68

c) Sinking fund schedule


No. of
payme
nts
1
2
3
4
5
6

Perio
dic
Depo
sit
3,00
0
3,000
3,000
3,000
3,000
3,000

Interest
in Fund

Increase
in Fund

Amount
in Fund

3,000.00

3,000.00

22.50
45.17
68.01
91.02
114.20

3,022.50
3,045.17
3,068.01
3,091.02
3,114.20

6,022.50
9,067.67
12,135.68
15,226.70
18,340.90

2. Three years from now, Mr. T needs 30,000 to liquidate a certain debt, at 6%
converted semi-annually.
a. How much must he deposit at the end of every 6 months to provide for the
payment of the debt?
b. Prepare a sinking fund table showing the growth of the fund for 3 years.
a. Given:

S = 30,000
j = 6% = .06
m=4

R=
R=

30,000(.05)
[ (1.015)61 ]
450
.093443263

R = 4,815.75

b. Sinking fund table


Payme
nts
1
2
3
4
5
6

Deposi
t
4,815.
76
4,815.7
6
4,815.7
6
4,815.7
6
4,815.7
6
4,815.7
6

Interest
0

Increase
in Fund
4,815.76

Amount
in Fund
4,815.76

72.24

4,888.00

9,703.76

145.56

4,961.32

14,665.08

219.98

5,035.74

19,700.82

295.51

5,111.27

24,812.09

372.18

5,187.94

30,000.03

Exercise 4.2
1. A fund is being created by paying 2,500 at the end of each year a 10.25%
compounded annually for 4 years.
a. How much money is in the fund just after the 3rd deposit?
b. Construct a sinking fund schedule for 4 years.
2. Mr. Imperial will deposit 10,000 at the QUAK Bank at the end of each
quarter for 2 years. If the banks gives out 9% compounded quarterly, find
the amount to his credit just after the last deposit.
3. What monthly payment into a sinking fund at 8% compounded semi-annually
will be needed to raise 50,200 at the end of 2 years and 6 months?
4. A fund of 50,500 is to be raised by 6 semi-annul payments at 6% converted
every 6 months.
a. Find the semi-annual payment
b. Construct a sinking fund table.
5. Prepare a sinking fund table that shows the growth of fund in 5 months if
1,300 is to be paid at the rate of 12% converted monthly.
4.3
Sinking Fund Method of Retiring a Debt
A typical loan of moderate size to an individual or a corporation is discharge by
the amortization method.

Illustrative Example
1. The principal of a loan 40,000 will be at the end of 15 years by the
accumulation of a sinking fund by quarterly deposits, and interest will be
payable on the debt quarterly at the rate of 9%.
a. Find the quarterly expense of the loan to debtor if his sinking fund is
invested at 8% compounded quarterly.
b. Find the book value of the debt just after the 4th deposit.
c. Find the quarterly expense of the debt if he discharge all the liability as to
principal and interest by paying his creditor equal sums at the end of each
3 months for 2 years.
Given:

S = 45,000
n=6

Solution:
R=

S (i)
[ (1+i)n 1 ]

R=

40,000(.02)
[ (1.02)n6 1 ]

R = 6,341.03

The interest payable quarterly to the creditor is 40,000(.0225) = 900.


Therefore, the quarterly expense of the debt is 6,341.03 + 900 = 7

You might also like